Cases from Everyday Neurology Practice
Quiz-summary
0 of 54 questions completed
Questions:
- 1
- 2
- 3
- 4
- 5
- 6
- 7
- 8
- 9
- 10
- 11
- 12
- 13
- 14
- 15
- 16
- 17
- 18
- 19
- 20
- 21
- 22
- 23
- 24
- 25
- 26
- 27
- 28
- 29
- 30
- 31
- 32
- 33
- 34
- 35
- 36
- 37
- 38
- 39
- 40
- 41
- 42
- 43
- 44
- 45
- 46
- 47
- 48
- 49
- 50
- 51
- 52
- 53
- 54
Information
This quiz contains neurological problems encountered by Professor Gates in his clinical practice. Most of the questions are multiple choice, once you have chosen the correct answer, an explanation will be provided before moving on to the next question.
Some cases extend over two questions.
You have already completed the quiz before. Hence you can not start it again.
Quiz is loading...
You must sign in or sign up to start the quiz.
You have to finish following quiz, to start this quiz:
Results
0 of 54 questions answered correctly
Your time:
Time has elapsed
You have reached 0 of 0 points, (0)
Categories
- Not categorized 0%
- Case 1 0%
- Case 10 0%
- Case 11 0%
- Case 12 0%
- Case 13 0%
- Case 14 0%
- Case 15 0%
- Case 16 0%
- Case 17 0%
- Case 18 0%
- Case 19 0%
- Case 2 0%
- Case 20 0%
- Case 21 0%
- Case 22 0%
- Case 23 0%
- Case 24 0%
- Case 25 0%
- Case 26 0%
- Case 27 0%
- Case 3 0%
- Case 30 0%
- Case 31 0%
- Case 32 0%
- Case 33 0%
- Case 34 0%
- Case 35 0%
- Case 36 0%
- Case 37 0%
- Case 38 0%
- Case 39 0%
- Case 4 0%
- Case 40 0%
- Case 41 0%
- Case 5 0%
- Case 6 0%
- Case 7 0%
- Case 8 0%
- Case 9 0%
- Mri vs Ultrasound 0%
- Question 28 0%
- 1
- 2
- 3
- 4
- 5
- 6
- 7
- 8
- 9
- 10
- 11
- 12
- 13
- 14
- 15
- 16
- 17
- 18
- 19
- 20
- 21
- 22
- 23
- 24
- 25
- 26
- 27
- 28
- 29
- 30
- 31
- 32
- 33
- 34
- 35
- 36
- 37
- 38
- 39
- 40
- 41
- 42
- 43
- 44
- 45
- 46
- 47
- 48
- 49
- 50
- 51
- 52
- 53
- 54
- Answered
- Review
-
Question 1 of 54
1. Question
Category: Case 1What is the single most important question to ask patients with headache?
Correct
Medical students and young doctors are taught many questions to ask about headache, most of these questions are not particularly helpful. Whether the headache is constant or throbbing rarely helps to differentiate the various causes of headache, the severity of pain is subjective so once again severity does not help differentiate the cause. The site of the headache is important, a focal headache or focal facial pain may indicate a specific diagnosis such as cluster headache that is always unilateral in or around the eye a as opposed to a generalised headache.
The single most important question to ask is the rapidity of onset from the moment it starts until it reaches its maximum intensity. This will help differentiate between the various causes of headache eg SAH, Meningitis, Migraine and a Hangover! All four conditions will present with headache, nausea, vomiting, photophobia, phonophobia and in some instances neck stiffness.
Incorrect
Medical students and young doctors are taught many questions to ask about headache, most of these questions are not particularly helpful. Whether the headache is constant or throbbing rarely helps to differentiate the various causes of headache, the severity of pain is subjective so once again severity does not help differentiate the cause. The site of the headache is important, a focal headache or focal facial pain may indicate a specific diagnosis such as cluster headache that is always unilateral in or around the eye a as opposed to a generalised headache.
The single most important question to ask is the rapidity of onset from the moment it starts until it reaches its maximum intensity. This will help differentiate between the various causes of headache eg SAH, Meningitis, Migraine and a Hangover! All four conditions will present with headache, nausea, vomiting, photophobia, phonophobia and in some instances neck stiffness.
-
Question 2 of 54
2. Question
Category: Case 2A 67 year-old male with 18 months of an altered sensation in front of both ears and a feeling of weakness in the legs, the left worse than the right.
What anatomical structures or pathways are affected? (More than one correct answer)
Correct
Correct Answers: Corticospinal Tract and Trigeminal Sensory Nucleus.
Weakness means the motor pathway is affected either in the peripheral nervous system or central nervous system somewhere between the muscle and the motor cortex.
The altered sensation in front of both ears is within the distribution of the trigeminal (5th cranial) nerve.
Incorrect
Correct Answers: Corticospinal Tract and Trigeminal Sensory Nucleus.
Weakness means the motor pathway is affected either in the peripheral nervous system or central nervous system somewhere between the muscle and the motor cortex.
The altered sensation in front of both ears is within the distribution of the trigeminal (5th cranial) nerve.
Hint
The way to solve this is to look up the anatomy.
-
Question 3 of 54
3. Question
Category: Case 2For the same 67 yo male from the last question, where would you place the problem in the nervous system?
Correct
The altered sensation in front of both ears is the clue and it is the parallel of latitude, but you need a little piece of neuroanatomy knowledge to realise this. The 5th cranial nerve dips down into the upper cervical cord with the lower most fibres representing sensation in front of both ears. An “onion skin” sensory loss spreading forward from the ears is classical for a lesion at the level of the foramen magnum such as a syrinx
One would very much suspect that he has a lesion at the level of the foramen magnum
The appropriate diagnostic test would be an MRI of his upper cervical spine and foramen magnum region.
Incorrect
The altered sensation in front of both ears is the clue and it is the parallel of latitude, but you need a little piece of neuroanatomy knowledge to realise this. The 5th cranial nerve dips down into the upper cervical cord with the lower most fibres representing sensation in front of both ears. An “onion skin” sensory loss spreading forward from the ears is classical for a lesion at the level of the foramen magnum such as a syrinx
One would very much suspect that he has a lesion at the level of the foramen magnum
The appropriate diagnostic test would be an MRI of his upper cervical spine and foramen magnum region.
-
Question 4 of 54
4. Question
Category: Case 2The MRI Scan was normal for this 67 yo male. What would you do next?
Correct
All Answers are correct!
There are several options when you are uncertain of the diagnosis. The most common employed is to adopt a wait and see approach, this is only ok if you are not dealing with a condition that will cause harm if not dealt with.
A 2nd approach is to obtain a second opinion.
A 3rd approach is the advice given to me by a mentor and outstanding clinical neurologist Dr Arthur Schwieger. One day I said to him “Arthur I have just seen a patient, taken a history and performed an examination and I don’t have a clue what is wrong, what do I do? His reply “Start again” Best advice ever given.
If the x-ray does not detect what you were expecting, then another option is to discuss with the radiologist tell them what you are looking for and make certain the x-ray was of good quality and that the correct area was imaged. You know the patient, sometimes this leads them seeing some thing had missed but more importantly you have confirmed that the correct imaging has been performed and the quality is good.
Incorrect
All Answers are correct!
There are several options when you are uncertain of the diagnosis. The most common employed is to adopt a wait and see approach, this is only ok if you are not dealing with a condition that will cause harm if not dealt with.
A 2nd approach is to obtain a second opinion.
A 3rd approach is the advice given to me by a mentor and outstanding clinical neurologist Dr Arthur Schwieger. One day I said to him “Arthur I have just seen a patient, taken a history and performed an examination and I don’t have a clue what is wrong, what do I do? His reply “Start again” Best advice ever given.
If the x-ray does not detect what you were expecting, then another option is to discuss with the radiologist tell them what you are looking for and make certain the x-ray was of good quality and that the correct area was imaged. You know the patient, sometimes this leads them seeing some thing had missed but more importantly you have confirmed that the correct imaging has been performed and the quality is good.
-
Question 5 of 54
5. Question
Category: Case 3A 52 year old man presents with 7 episodes of transient left sided sensory symptoms spreading slowly from the back of his left forearm to the left side of his trunk and leg , the sensory disturbance spread over 5-10 minutes, it lasts about 20 minutes and as it subsided he developed a right-sided headache that increased rapidly to 9/10 severity. One episode was associated with nausea and vomiting.
These episodes were occurring every two or three days.
There was no significant past or family history and he was not on any medications.
What is your diagnosis and what is the clue to the diagnosis?
Correct
Each individual episode is clearly migraine because the initial symptoms are resolving before the latter symptoms are appearing or worsening. No other condition other than migraine has this characteristic mode of onset.
However this case highlights a second principle and that is if there is a change in the tempo of an illness you should always think of an alternative explanation or diagnosis.
The unusual aspect of this case is the flurry of events every second day out of the blue in a man who’s never had migraine in the past and there is no family history of migraine.
Incorrect
Each individual episode is clearly migraine because the initial symptoms are resolving before the latter symptoms are appearing or worsening. No other condition other than migraine has this characteristic mode of onset.
However this case highlights a second principle and that is if there is a change in the tempo of an illness you should always think of an alternative explanation or diagnosis.
The unusual aspect of this case is the flurry of events every second day out of the blue in a man who’s never had migraine in the past and there is no family history of migraine.
-
Question 6 of 54
6. Question
Category: Case 3Continuing from the last question, what diagnostic procedure was performed?
Correct
The diagnostic procedure was a lumbar puncture, it revealed 45 white cells all lymphocytes, a normal protein, a normal glucose and an opening pressure of 32 cm H2O (normal less than 20 cm H2O).
Explanation
This man has a very rare form of migraine referred to as HaNDL; this is an acronym for Headache, Neurological Deficit Lymphocytosis in the CSF. The aetiology of this condition is unknown, it is a self-limiting condition although some case reports suggest that the duration may be shortened by a course the corticosteroids. I do not want to teach you about rare diseases but I wanted to emphasise the principles involved that this case highlights.
His symptoms resolved spontaneously.
Formal Cerebral Angiography has largely been abandoned with the Advent of CT Angiography and MR Angiography. It is still used in the treatment of aneurysms.
Tests rarely help with intermittent disturbances and in this case A contrast CT brain and an MRI brain would both have been normal.
Incorrect
The diagnostic procedure was a lumbar puncture, it revealed 45 white cells all lymphocytes, a normal protein, a normal glucose and an opening pressure of 32 cm H2O (normal less than 20 cm H2O).
Explanation
This man has a very rare form of migraine referred to as HaNDL; this is an acronym for Headache, Neurological Deficit Lymphocytosis in the CSF. The aetiology of this condition is unknown, it is a self-limiting condition although some case reports suggest that the duration may be shortened by a course the corticosteroids. I do not want to teach you about rare diseases but I wanted to emphasise the principles involved that this case highlights.
His symptoms resolved spontaneously.
Formal Cerebral Angiography has largely been abandoned with the Advent of CT Angiography and MR Angiography. It is still used in the treatment of aneurysms.
Tests rarely help with intermittent disturbances and in this case A contrast CT brain and an MRI brain would both have been normal.
-
Question 7 of 54
7. Question
Category: Case 4A 65-year-old man came to hospital with 3 months of worsening dysarthria, dysphagia for both solids and fluids, and significant weight loss. Up until the time of admission he had not aspirated. Initially there had been double vision but this had resolved.
He noticed his symptoms were less severe first thing in the morning when he woke up and he also noticed that he could speak more clearly if he lay on his back with his head extended.
The examination revealed weakness of neck flexion and neck extension, bilateral facial weakness affecting all muscles of the face, poor excursion of the palate, normal gag reflexes, slow movements of the tongue and his speech demonstrated a nasal escape. There was no weakness, wasting, fasciculations or reflex abnormalities in the limbs and sensation was normal.
What part of the nervous system do you think is affected?
Correct
Lesion in the brainstem could produce dysarthria, dysphagia, the incompetent soft palate, possible facial weakness and double vision, however he would have to involve the pons and the medulla without producing impaired consciousness and affecting the limbs – most unlikely. Furthermore, it would not produce neck flexion and neck extension weakness. Anterior horn cell disease i.e. motor neurone disease could produce all these symptoms but the absence of fasciculations makes that diagnosis less likely. Spinal cord compression at the level of foramen magnum, this is where the spinal cord meets the brainstem, cannot produce dysarthria and dysphagia. Clearly this must be Myasthenia Gravis with the problem at the neuromuscular junction
Incorrect
Lesion in the brainstem could produce dysarthria, dysphagia, the incompetent soft palate, possible facial weakness and double vision, however he would have to involve the pons and the medulla without producing impaired consciousness and affecting the limbs – most unlikely. Furthermore, it would not produce neck flexion and neck extension weakness. Anterior horn cell disease i.e. motor neurone disease could produce all these symptoms but the absence of fasciculations makes that diagnosis less likely. Spinal cord compression at the level of foramen magnum, this is where the spinal cord meets the brainstem, cannot produce dysarthria and dysphagia. Clearly this must be Myasthenia Gravis with the problem at the neuromuscular junction
-
Question 8 of 54
8. Question
Category: Case 4What is your diagnosis for the previous question?
Correct
The fluctuating symptoms and the signs clearly point to the diagnosis of myasthenia gravis
Incorrect
The fluctuating symptoms and the signs clearly point to the diagnosis of myasthenia gravis
-
Question 9 of 54
9. Question
Category: Case 5This case highlights what to do when you have never seen the problem before.
The patient and her husband said I would be a miracle worker if I could help, what would you do to become a miracle worker?
A 74-year-old woman presents with a clicking noise in her left ear. It has been present for approximately 6 months, she had consulted several doctors who either told she that there was nothing wrong or that she would have to live with it.
Initially there were two brief episodes of true vertigo with the room spinning, these were separated by two weeks. The first one occurred when she hopped out of bed and the second when she bent over.
After these 2 episodes she developed the clicking noise that commenced as a soft, slow repetitive clicking. It may only click for a few times and then subside. Usually it increases in frequency and loudness over about 20 seconds and then subsided after a sudden whooshing sensation in the head making her feel vaguely unwell. This could sometimes be precipitated by a loud noise, for example, the vacuum cleaner, but often it occurred spontaneously. These episodes occurred many times per day.
An MRI scan showed diffuse white-matter disease.
Eardrops failed to help, she was given stillnox to try and help her but she couldn’t tolerate it and was then given restavit which also was without benefit.
The examination revealed very mild deafness in the left ear with webers sign (tuning fork in the middle of the forehead) localising to the left, and yet air conduction was greater than bone conduction bilaterally! I.e. the signs didn’t make sense
This is the sort of challenge I love what would you do?
Correct
Explanation:
The challenge of this case is a situation when you see someone with symptoms that you either have never seen before or that you do not understand. There is a tendency for a number of doctors to say that the patient is psychiatrically unwell. There is also a tendency for other doctors just to give up and say they don’t know, such was the case here. She had consulted three ENT surgeons. So ringing an ENT surgeon would not be an option.
Brief stereotyped symptoms indicate an organic pathology, prolonged symptoms that fluctuate both in intensity and content are more suggestive of a non-organic problems.
When I see someone with symptoms I have never seen before I simply go to the Internet i.e consult Dr Google. There is no excuse for not looking things up. In fact, if you just type in repetitive clicking noises in the ear, you will be a miracle worker and find the explanation. There are 2 causes; one is palatial myoclonus and this was not present in this patient. The other is middle ear myoclonus, if it affects the tensor Tympani muscle and the result is a repetitive vibration noise. However, if it affects the Stapedius muscle the result is a repetitive clicking noise.
The patient’s eustachian tube was blocked. Although I was doubtful that this could be the cause, the first step I took was to get her to unblock the eustachian tube with a nasal decongestant. To do this the patient has to look down at the floor and then hold the decongestant spray vertically in order to directed towards the Ostium of the Eustachian tube. This failed to lead to any resolution of the problem.
Anticonvulsants can be used to treat this or it may require surgery.
Incorrect
Explanation:
The challenge of this case is a situation when you see someone with symptoms that you either have never seen before or that you do not understand. There is a tendency for a number of doctors to say that the patient is psychiatrically unwell. There is also a tendency for other doctors just to give up and say they don’t know, such was the case here. She had consulted three ENT surgeons. So ringing an ENT surgeon would not be an option.
Brief stereotyped symptoms indicate an organic pathology, prolonged symptoms that fluctuate both in intensity and content are more suggestive of a non-organic problems.
When I see someone with symptoms I have never seen before I simply go to the Internet i.e consult Dr Google. There is no excuse for not looking things up. In fact, if you just type in repetitive clicking noises in the ear, you will be a miracle worker and find the explanation. There are 2 causes; one is palatial myoclonus and this was not present in this patient. The other is middle ear myoclonus, if it affects the tensor Tympani muscle and the result is a repetitive vibration noise. However, if it affects the Stapedius muscle the result is a repetitive clicking noise.
The patient’s eustachian tube was blocked. Although I was doubtful that this could be the cause, the first step I took was to get her to unblock the eustachian tube with a nasal decongestant. To do this the patient has to look down at the floor and then hold the decongestant spray vertically in order to directed towards the Ostium of the Eustachian tube. This failed to lead to any resolution of the problem.
Anticonvulsants can be used to treat this or it may require surgery.
-
Question 10 of 54
10. Question
Category: Case 6A 56-year-old man is sent for nerve conduction studies. He has permanent numbness in the second and third digits of the right hand and is suspected of having carpal tunnel syndrome. The nerve conduction study does indeed confirm a very mild carpal tunnel syndrome.
What would you do next?
Correct
Explanation:
Carpal tunnel syndrome is one of the most common neurological problems and yet it seems to be poorly understood by many. The pathognomonic symptom is being woken at night with numbness and/or in the hand(s) that is relieved by hanging the hand out of the side of the bed or moving the hand. It is not uncommon for the condition to be present in both hands. Similar numbness and/or pins and needles can occur when the patient is holding objects, for example – a book, riding a bicycle, driving the car, knitting or mowing the lawns.
Patients do not read textbooks and sometimes their symptoms appear to be atypical. It is not uncommon for patients to feel that the whole hand is affected. It is not uncommon for patients to complain of pain. However, a word of warning “if pain occurs when there are no sensory symptoms present, that pain is very unlikely to be related to carpal tunnel syndrome and therefore very unlikely to resolve with treatment to the carpal tunnel syndrome.” This is the most common reason for failed carpal tunnel surgery, symptoms that were never related to carpal tunnel persist following the operation. Some patients even complain of pain going up the arm.
Interpreting nerve conduction studies in carpal tunnel syndrome
The symptoms of carpal tunnel syndrome are intermittent and, like any intermittent problem, often tests for that problem can be normal. I explain to the patient that the symptoms of carpal tunnel syndrome are due to irritation of the nerve whereas the nerve conduction studies can only detect pressure and damage to the nerve.
Therefore a normal nerve conduction study does not exclude very mild or recent onset carpal tunnel syndrome. It is also not uncommon for patients without symptoms have evidence of median nerve compression at the wrist on nerve conduction studies. In older patients, it is not uncommon for them to present with few symptoms and severe changes on the nerve conduction studies.
The earliest change seen is a mild prolongation of the sensory potential except in the very rare instance of someone with a pure motor carpal tunnel syndrome. As the degree of compression worsens, the sensory latency will be further delayed and the patient will develop a prolongation of the distal motor latency which also increases with worsening severity. Initially, it may only be prolonged compared to the corresponding distal latency of the ulnar nerve but subsequently it is prolonged in absolute terms.
Permanent numbness and weakness only occurs with more severe compression and therefore more severe findings on the nerve conduction studies. In this patient, the findings were only mild and therefore could not be the explanation for permanent numbness. Once again, one needs to define the exact nature and distribution of the symptoms. This man had tingling on the back of the hand between the first and second digits radiating up the lateral border of the right forearm. The distribution was in that of the C6 nerve root. His Brachioradialis reflex was absent, strength was normal. He clearly had a painless sensory C6 Radiculopathy.
Thus the answer is question the diagnosis of carpal tunnel as the cause for his symptoms.
Incorrect
Explanation:
Carpal tunnel syndrome is one of the most common neurological problems and yet it seems to be poorly understood by many. The pathognomonic symptom is being woken at night with numbness and/or in the hand(s) that is relieved by hanging the hand out of the side of the bed or moving the hand. It is not uncommon for the condition to be present in both hands. Similar numbness and/or pins and needles can occur when the patient is holding objects, for example – a book, riding a bicycle, driving the car, knitting or mowing the lawns.
Patients do not read textbooks and sometimes their symptoms appear to be atypical. It is not uncommon for patients to feel that the whole hand is affected. It is not uncommon for patients to complain of pain. However, a word of warning “if pain occurs when there are no sensory symptoms present, that pain is very unlikely to be related to carpal tunnel syndrome and therefore very unlikely to resolve with treatment to the carpal tunnel syndrome.” This is the most common reason for failed carpal tunnel surgery, symptoms that were never related to carpal tunnel persist following the operation. Some patients even complain of pain going up the arm.
Interpreting nerve conduction studies in carpal tunnel syndrome
The symptoms of carpal tunnel syndrome are intermittent and, like any intermittent problem, often tests for that problem can be normal. I explain to the patient that the symptoms of carpal tunnel syndrome are due to irritation of the nerve whereas the nerve conduction studies can only detect pressure and damage to the nerve.
Therefore a normal nerve conduction study does not exclude very mild or recent onset carpal tunnel syndrome. It is also not uncommon for patients without symptoms have evidence of median nerve compression at the wrist on nerve conduction studies. In older patients, it is not uncommon for them to present with few symptoms and severe changes on the nerve conduction studies.
The earliest change seen is a mild prolongation of the sensory potential except in the very rare instance of someone with a pure motor carpal tunnel syndrome. As the degree of compression worsens, the sensory latency will be further delayed and the patient will develop a prolongation of the distal motor latency which also increases with worsening severity. Initially, it may only be prolonged compared to the corresponding distal latency of the ulnar nerve but subsequently it is prolonged in absolute terms.
Permanent numbness and weakness only occurs with more severe compression and therefore more severe findings on the nerve conduction studies. In this patient, the findings were only mild and therefore could not be the explanation for permanent numbness. Once again, one needs to define the exact nature and distribution of the symptoms. This man had tingling on the back of the hand between the first and second digits radiating up the lateral border of the right forearm. The distribution was in that of the C6 nerve root. His Brachioradialis reflex was absent, strength was normal. He clearly had a painless sensory C6 Radiculopathy.
Thus the answer is question the diagnosis of carpal tunnel as the cause for his symptoms.
-
Question 11 of 54
11. Question
Category: Case 7A 38-year-old man suffers a low speed motor vehicle accident when his car veered into the left-hand barriers. There was a minor bump to his head but he has no recollection of the accident.
Later, in the ambulance and in the Emergency Department, he had trembling of his upper and lower limbs that lasted on-off initially for two hours, subsided for another hour or two, only to recur and lasting another hour.. He was completely lucid during these episodes, he noticed some difficulty with his breathing.
A CT scan of the brain demonstrated significant bilateral frontal lobe damage from a previous major head injury with an old fractured skull.
What is your diagnosis?
Correct
Explanation:
The principle here is that seizures are brief, stereotyped, with positive phenomena (no loss of function), with or without a fall and or LOC. His episodes do not fit this definition.
Fluctuating symptoms, particularly if associated with symptoms outside the nervous system such as difficulty breathing or chest discomfort, are not primarily neurological in origin. The trembling in the Emergency Department and ambulance was hyperventilation syndrome with reproduction of his symptoms he was made to hyperventilate.
It was assumed that the if accident had occurred as the result of a seizure, that the seizures were caused by his extensive brain damage from previous head injury. The questions to ask are: has the patient had anything to suggest minor seizures? Such as if the patient has ever had any unexplained motor vehicle accidents. Ask if they miss parts of television programmes or conversations suggesting they switched off. Ask if they have been called a daydreamer and then ask family members if they have seen any episodes in which “the lights are on but there is no one home”. These would all point to clinically unrecognised absence or complex partial seizures. Remember simple partial seizures refer to those that do not involve loss of awareness whereas complex partial seizures (which used to be called temporal lobe epilepsy) are where the patient has impaired awareness of what is happening around them and is unresponsive for a short period of time often with a short period of postictal confusion.
The cause of the accident?
An eyewitness was spoken to and they saw him slump at the wheel, there was no evidence of a convulsion. He had been very sleep deprived and recalled being very drowsy, his car was witnessed to slowly veer across to the left side of the road before it crashed. He had been up most of the night before and had very little sleep, clearly he fell asleep at the wheel. Although another possibility is that he had an arrhythmia causing him to fall asleep at the wheel. He was monitored for 48 hours in the Emergency Department and no arrhythmia was detected, but this doesn’t exclude an arrhythmia.
Incorrect
Explanation:
The principle here is that seizures are brief, stereotyped, with positive phenomena (no loss of function), with or without a fall and or LOC. His episodes do not fit this definition.
Fluctuating symptoms, particularly if associated with symptoms outside the nervous system such as difficulty breathing or chest discomfort, are not primarily neurological in origin. The trembling in the Emergency Department and ambulance was hyperventilation syndrome with reproduction of his symptoms he was made to hyperventilate.
It was assumed that the if accident had occurred as the result of a seizure, that the seizures were caused by his extensive brain damage from previous head injury. The questions to ask are: has the patient had anything to suggest minor seizures? Such as if the patient has ever had any unexplained motor vehicle accidents. Ask if they miss parts of television programmes or conversations suggesting they switched off. Ask if they have been called a daydreamer and then ask family members if they have seen any episodes in which “the lights are on but there is no one home”. These would all point to clinically unrecognised absence or complex partial seizures. Remember simple partial seizures refer to those that do not involve loss of awareness whereas complex partial seizures (which used to be called temporal lobe epilepsy) are where the patient has impaired awareness of what is happening around them and is unresponsive for a short period of time often with a short period of postictal confusion.
The cause of the accident?
An eyewitness was spoken to and they saw him slump at the wheel, there was no evidence of a convulsion. He had been very sleep deprived and recalled being very drowsy, his car was witnessed to slowly veer across to the left side of the road before it crashed. He had been up most of the night before and had very little sleep, clearly he fell asleep at the wheel. Although another possibility is that he had an arrhythmia causing him to fall asleep at the wheel. He was monitored for 48 hours in the Emergency Department and no arrhythmia was detected, but this doesn’t exclude an arrhythmia.
-
Question 12 of 54
12. Question
Category: Case 8A 61-year-old man was seen with a 46 year history of funny turns.
These episodes occurred 3-4 times per year, as often as 6 or 7 times in recent years.
The initial symptom is a feeling of something rising from his groin half way up his stomach or to his lower chest. This can recur 2-3 times before an episode rises from his groin, travels all the way up through his head whereupon he blacks out.
When he blacks out he falls to the ground or slumps in the chair. His wife has seen him fall to the ground with one of these episodes, have a few brief jerks, and then wake up immediately as if nothing had happened.
He has never bitten his tongue or wet his pants.
He has had 2 episodes whilst driving and on both occasions he had sufficient time to stop and park the car on the side of the road.
He is otherwise fit and well, a non-smoker and drinks four full strength stubbies of beer a day (8 glasses).
On examination the pulse was 74 and regular and his blood pressure was 140/90 lying and sitting, the neurological examination was completely normal.
Your diagnosis?
Correct
Explanation:
If you thought he might have epilepsy you would have agreed with the doctors who have seen him for the previous 46 years. However, you were wrong. Although a rising sensation is often a typical Aura of a seizure, if this was the aura of a tonic-clonic seizure, then he should have actually had convulsions and he didn’t. If this was a complex partial seizure he would have blackouts, but he wouldn’t have fallen to the ground or slumped in the chair. If it were a seizure, he wouldn’t wake up immediately and be perfectly normal even after a few brief jerks. If it were a simple partial seizure, he would not blackout and he would not fall. The brief loss of consciousness with rapid return to normal indicates almost certainly a Cardiac Arrhythmia. An implantable loop recorder was inserted and within one month 3 episodes of complete heart block were documented. His “seizures” resolved following insertion of a pacemaker.
Incorrect
Explanation:
If you thought he might have epilepsy you would have agreed with the doctors who have seen him for the previous 46 years. However, you were wrong. Although a rising sensation is often a typical Aura of a seizure, if this was the aura of a tonic-clonic seizure, then he should have actually had convulsions and he didn’t. If this was a complex partial seizure he would have blackouts, but he wouldn’t have fallen to the ground or slumped in the chair. If it were a seizure, he wouldn’t wake up immediately and be perfectly normal even after a few brief jerks. If it were a simple partial seizure, he would not blackout and he would not fall. The brief loss of consciousness with rapid return to normal indicates almost certainly a Cardiac Arrhythmia. An implantable loop recorder was inserted and within one month 3 episodes of complete heart block were documented. His “seizures” resolved following insertion of a pacemaker.
-
Question 13 of 54
13. Question
Category: Case 9A 77-year-old man is referred for nerve conduction studies for suspected ulnar nerve lesion with intermittent pain and tingling in his right hand, the tingling affects the fifth digit. The nerve conduction studies were completely normal.
Diagnosis?
Correct
Explanation:
Take a proper history and perform a detailed and directed examination! Ask the patient exactly where the pins and needles are, in this case they were affecting the palm and dorsal aspect of the fifth finger. That could be an ulnar nerve lesion but it could also be a C8 radiculopathy. Both can cause pain although pain is much more common with a radiculopathy. Palpation of the ulnar nerve in the groove at the elbow reproduced the tingling and pain. This man was resting his elbow on hard surfaces and irritating his ulnar nerve. The principal here is that symptoms are due to irritation of the nerve whilst the nerve conduction studies can only detect problems when there is pressure and damage to the nerve. This is particularly common in carpal tunnel syndrome and ulnar nerve lesions. The positive Tinel’s sign at the elbow clearly indicates that this is an ulnar nerve lesion at that site, he was advised to avoid resting his arm on the other surfaces.
Incorrect
Explanation:
Take a proper history and perform a detailed and directed examination! Ask the patient exactly where the pins and needles are, in this case they were affecting the palm and dorsal aspect of the fifth finger. That could be an ulnar nerve lesion but it could also be a C8 radiculopathy. Both can cause pain although pain is much more common with a radiculopathy. Palpation of the ulnar nerve in the groove at the elbow reproduced the tingling and pain. This man was resting his elbow on hard surfaces and irritating his ulnar nerve. The principal here is that symptoms are due to irritation of the nerve whilst the nerve conduction studies can only detect problems when there is pressure and damage to the nerve. This is particularly common in carpal tunnel syndrome and ulnar nerve lesions. The positive Tinel’s sign at the elbow clearly indicates that this is an ulnar nerve lesion at that site, he was advised to avoid resting his arm on the other surfaces.
-
Question 14 of 54
14. Question
Category: Case 10A 65-year-old man noticed mild pain in his left upper lateral thigh towards the end of the game of golf. There had not been any obvious injury. The pain was present at rest without any movement. Over the next 48 hours the pain increased in severity such that he could not get in or out of a chair or bed and and he had difficulty walking because of the pain. Thus the pain was made worse by movement of the limb.
What is your diagnosis?
Correct
A clinical diagnosis of trochanteric bursitis was made and treatment was recommended with rest and ice packs. The first ice-pack was applied on day two, it lead to increased pain and stiffness and increased difficulty walking. The patient was encouraged to persevere with that ice-pack, reluctantly a second ice-pack was applied and the result was exactly the same.
Continue to the next question.
Incorrect
A clinical diagnosis of trochanteric bursitis was made and treatment was recommended with rest and ice packs. The first ice-pack was applied on day two, it lead to increased pain and stiffness and increased difficulty walking. The patient was encouraged to persevere with that ice-pack, reluctantly a second ice-pack was applied and the result was exactly the same.
Continue to the next question.
-
Question 15 of 54
15. Question
Category: Case 10For the same 65-year-old man, what principle applies in this case?
Correct
The principle is that if a treatment does not work then it is either:
Wrong Treatment
Wrong Dose
Wrong Diagnosis
Incorrect
The principle is that if a treatment does not work then it is either:
Wrong Treatment
Wrong Dose
Wrong Diagnosis
-
Question 16 of 54
16. Question
Category: Case 10A clinical diagnosis of trochanteric bursitis was made and treatment was recommended with rest and ice packs. The first icepack was applied on day two, it lead to increased pain and stiffness and increased difficulty walking. The patient was encouraged to persevere with that icepack, reluctantly a second icepack was applied and the result was exactly the same.
A diagnostic procedure was performed. What was it?
Correct
The diagnostic test was an ultrasound of the thigh that demonstrated calcific myositis, a very rare condition which usually follows an injury to the muscle and the formation of a haematoma. The patient develops infiltration of calcium within the haematoma in the muscle causing severe pain. Steroids and local anaesthetic were injected into the site of calcium formation with significant but temporary relief of pain.
The diagnosis was correct but there was a possibility of an osteosarcoma and therefore a plain x-ray of the femur was performed to ensure that the calcium was in fact confined to the muscle. This revealed isolated fluffy calcium confined to the Vastus Lateralis muscle.
A second injection of a higher dose of steroids and local anaesthetic was given with complete relief of symptoms.
I was able to return to my golf and bike riding. Yes, this was me!
Incorrect
The diagnostic test was an ultrasound of the thigh that demonstrated calcific myositis, a very rare condition which usually follows an injury to the muscle and the formation of a haematoma. The patient develops infiltration of calcium within the haematoma in the muscle causing severe pain. Steroids and local anaesthetic were injected into the site of calcium formation with significant but temporary relief of pain.
The diagnosis was correct but there was a possibility of an osteosarcoma and therefore a plain x-ray of the femur was performed to ensure that the calcium was in fact confined to the muscle. This revealed isolated fluffy calcium confined to the Vastus Lateralis muscle.
A second injection of a higher dose of steroids and local anaesthetic was given with complete relief of symptoms.
I was able to return to my golf and bike riding. Yes, this was me!
-
Question 17 of 54
17. Question
Category: Case 11A 45-year-old man presents with severe pain in his right upper arm, associated weakness in his hand and numbness affecting the medial aspect of the forearm but not affecting his hand. An x-ray of his cervical spine reveals significant degenerative disease with foraminal stenoses at most levels.
The examination revealed weakness of muscle number 3 and 4 in the hand, muscle number 1, 2 and 5 in the hand were normal. Muscles around the shoulder and elbow were normal. There was impaired sensation affecting the medial aspect of the forearm both on the volar and to a lesser extent dorsal surface extending from above the wrist to just above the elbow. You need to consult the table that I have given you on the muscles around the hand and you will be able to diagnose the cause of that pattern of weakness. You will need to look up an anatomy textbook (just as I did when I was examining this patient) to see what nerve is causing the sensory loss.
One other muscle in the hand was weak but it is not part of the 5*3*5 rule. yYou should be able to name that muscle if you look up an anatomy textbook. Involvement of muscles 3 and 4 indicate what nerve is affected and where the lesion is likely to be.
What is your diagnosis?
Correct
The first 3 of these diagnoses are possible with the history that you were given, each can cause pain, weakness and sensory loss. A pronator syndrome can cause weakness, rarely it can be painful but it does not cause sensory symptoms.
However the pattern of weakness affecting the 3rd muscle abductor pollicis brevis and 4th muscle the long flexors of the lateral two digits (flexor digitorum profundus innervated by the median not the ulnar nerve). The pattern of weakness pointed to a proximal median nerve lesion. This is the sort of weakness you would see with a pronator syndrome, but as we have said it shouldn’t produce sensory loss. The sensory loss is due to involvement of the medial cutaneous nerve of the forearm and this is close to the median nerve in the upper arm at the insertion of coracobrachialis muscle several centimetres above the elbow. Initial nerve conduction studies revealed a marked reduction in the amplitude of the median response of abductor pollicis brevis with normal sensory findings in the hands and normal conduction velocity in the median nerve. All other nerves were normal, although the medial cutaneous nerve of the forearm was not tested in the first study. Electromyography was also not performed during the first study because the symptoms had only been present for five days and denervation take somewhere between 10 days and three weeks to appear.
Incorrect
The first 3 of these diagnoses are possible with the history that you were given, each can cause pain, weakness and sensory loss. A pronator syndrome can cause weakness, rarely it can be painful but it does not cause sensory symptoms.
However the pattern of weakness affecting the 3rd muscle abductor pollicis brevis and 4th muscle the long flexors of the lateral two digits (flexor digitorum profundus innervated by the median not the ulnar nerve). The pattern of weakness pointed to a proximal median nerve lesion. This is the sort of weakness you would see with a pronator syndrome, but as we have said it shouldn’t produce sensory loss. The sensory loss is due to involvement of the medial cutaneous nerve of the forearm and this is close to the median nerve in the upper arm at the insertion of coracobrachialis muscle several centimetres above the elbow. Initial nerve conduction studies revealed a marked reduction in the amplitude of the median response of abductor pollicis brevis with normal sensory findings in the hands and normal conduction velocity in the median nerve. All other nerves were normal, although the medial cutaneous nerve of the forearm was not tested in the first study. Electromyography was also not performed during the first study because the symptoms had only been present for five days and denervation take somewhere between 10 days and three weeks to appear.
-
Question 18 of 54
18. Question
Category: Case 12A 50-year-old man presents with several months of pain on the right side of the neck radiating to the tip of the shoulder, up the side of the neck to the base of skull. During this same period of time he had noticed weakness in his right hand and altered feeling in the fourth and fifth digits of the hand.
He was suspected of having cervical nerve root compression. An MRI scan demonstrated severe foraminal stenosis at the C5-6 level bilaterally. He underwent decompressive surgery to the right C6 nerve root.
What is your diagnosis?
Correct
The distribution of his pain was beyond that of a single nerve root and followed the line of the right trapezius muscle. In fact this muscle was very tender to palpation. Thus, his neck pain was due to spasm of the trapezius muscle, this cannot explain the numbness in his hand, however, so there must be two problems.
He did not have pain radiating down the lateral aspect of the forearm that one would expect to see with a C6 radiculopathy. There is no way that a C6 nerve root compression could produce the sensory symptoms that this man had in the fourth and fifth digits of the right hand, that is either the ulnar nerve or a C8-21 nerve root problem.
A diagnostic procedure was performed, it was nerve conduction studies that demonstrated a straightforward ulnar nerve lesion at the elbow and, using Kimura’s inching technique, the site of compression was just at the medial epicondyle. The patient had a positive Tinel’s sign with palpation of the ulnar nerve at the elbow.
Incorrect
The distribution of his pain was beyond that of a single nerve root and followed the line of the right trapezius muscle. In fact this muscle was very tender to palpation. Thus, his neck pain was due to spasm of the trapezius muscle, this cannot explain the numbness in his hand, however, so there must be two problems.
He did not have pain radiating down the lateral aspect of the forearm that one would expect to see with a C6 radiculopathy. There is no way that a C6 nerve root compression could produce the sensory symptoms that this man had in the fourth and fifth digits of the right hand, that is either the ulnar nerve or a C8-21 nerve root problem.
A diagnostic procedure was performed, it was nerve conduction studies that demonstrated a straightforward ulnar nerve lesion at the elbow and, using Kimura’s inching technique, the site of compression was just at the medial epicondyle. The patient had a positive Tinel’s sign with palpation of the ulnar nerve at the elbow.
-
Question 19 of 54
19. Question
Category: Case 13Two months prior to being seen, a 29 year old man, having retired to bed without any problems, awoke with severe pain from his heel up to his right calf. The pain was so severe he felt nauseated, walked to the bathroom where he was witnessed to faint. The pain was so severe he could not walk. He attended the emergency department, they performed an ultrasound and then requested a formal ultrasound of the calf in the radiology department. These were both normal. He was discharged home without a diagnosis and advised to rest. The pain continued despite rest and five weeks after onset he had a repeat ultrasound of his right calf that demonstrated deep vein thrombosis. He was placed on warfarin. His pain continued and he noticed that he couldn’t walk properly because there was something wrong with his right leg, it wasn’t just the pain.
On examination, he was unable to stand on the toes of his right foot when only using the right foot but he could stand on the toes of both feet simultaneously. All reflexes were preserved, the left ankle reflex was perhaps a little reduced, this was not certain. There was no sensory deficit. There was no tenderness on palpation of the calf. Squeezing his calf muscle on the asymptomatic side resulted in flexion of the ankle, it failed to flex on the left. Whilst the patient was lying prone on the bed the asymptomatic ankle could be seen to be plantar flexed but the symptomatic side did not.
What is your diagnosis?
Correct
If you chose an S1 radiculopathy that could explain the onset of severe pain whilst in bed but once again this is very unusual.The preserved ankle reflex would be atypical for an S1 radiculopathy.
I was uncertain so if you chose uncertain that is a reasonable answer.
The patient should probably not have been sent home from the emergency department without diagnosis when the pain was so severe. The treating doctors were told the ultrasound of his calf was completely normal so they hadn’t found the explanation for his pain.
When I saw him, I thought he must’ve had a ruptured Achilles tendon, but why would this happen in the middle of the night when he is not weight-bearing?
The first thing that I did was to check that the original ultrasound did not reveal deep vein thrombosis by ringing the radiology department, they checked and were absolutely convinced that the scan was of excellent quality, had examined the correct area and was normal. However they only scanned the calf. I then checked the sensitivity and specificity of ultrasonography in the detection of DVT. Ultrasonography is considered to be the best non-invasive diagnostic method for DVT showing an average sensitivity and specificity of 97% for proximal deep vein thrombosis. However, ultrasonography cannot be relied on to diagnose calf vein thrombosis, the sensitivity for symptomatic calf vein thrombosis has been reported to be as low as 75%. Thus, it is conceivable that the first scan did not detect a deep vein thrombosis in the calf. However, I was still uneasy because such severe pain with a DVT is very unusual.
An inability to stand on the toes suggests a problem with the Achilles tendon or the gastrocnemius and soleus muscles. The two diagnostic tests performed at the bedside were tests that I had never heard of before. I simply looked them up on the Internet. As I suspected he may have ruptured his Achilles tendon, I looked for bedside tests. The first was the calf squeeze or Thompson’s test. In a normal person when you squeeze the calf the foot plantar flexes, if there is a rupture of the Achilles tendon the foot does not plantar flex. His did not plantar flex. The 2nd test, the knee flexion test or Matles test, is where the patient lies prone on the bedside and bends their knees 90° from the horizontal, in a normal person the sole of the foot flexes at the ankle and the toes are above the heel. In this man his right foot remained horizontal. Thus the two bedside tests suggested the probability of torn Achilles tendon. I discussed the appropriate radiological procedure with a radiologist and they recommend initially an ultrasound of the Achilles tendon and if that was unhelpful then an MRI scan.
The ultrasound was completely normal! There was no evidence of an Achilles tendon tear!
A further diagnostic procedure was requested. Continue to the next question.
Incorrect
If you chose an S1 radiculopathy that could explain the onset of severe pain whilst in bed but once again this is very unusual.The preserved ankle reflex would be atypical for an S1 radiculopathy.
I was uncertain so if you chose uncertain that is a reasonable answer.
The patient should probably not have been sent home from the emergency department without diagnosis when the pain was so severe. The treating doctors were told the ultrasound of his calf was completely normal so they hadn’t found the explanation for his pain.
When I saw him, I thought he must’ve had a ruptured Achilles tendon, but why would this happen in the middle of the night when he is not weight-bearing?
The first thing that I did was to check that the original ultrasound did not reveal deep vein thrombosis by ringing the radiology department, they checked and were absolutely convinced that the scan was of excellent quality, had examined the correct area and was normal. However they only scanned the calf. I then checked the sensitivity and specificity of ultrasonography in the detection of DVT. Ultrasonography is considered to be the best non-invasive diagnostic method for DVT showing an average sensitivity and specificity of 97% for proximal deep vein thrombosis. However, ultrasonography cannot be relied on to diagnose calf vein thrombosis, the sensitivity for symptomatic calf vein thrombosis has been reported to be as low as 75%. Thus, it is conceivable that the first scan did not detect a deep vein thrombosis in the calf. However, I was still uneasy because such severe pain with a DVT is very unusual.
An inability to stand on the toes suggests a problem with the Achilles tendon or the gastrocnemius and soleus muscles. The two diagnostic tests performed at the bedside were tests that I had never heard of before. I simply looked them up on the Internet. As I suspected he may have ruptured his Achilles tendon, I looked for bedside tests. The first was the calf squeeze or Thompson’s test. In a normal person when you squeeze the calf the foot plantar flexes, if there is a rupture of the Achilles tendon the foot does not plantar flex. His did not plantar flex. The 2nd test, the knee flexion test or Matles test, is where the patient lies prone on the bedside and bends their knees 90° from the horizontal, in a normal person the sole of the foot flexes at the ankle and the toes are above the heel. In this man his right foot remained horizontal. Thus the two bedside tests suggested the probability of torn Achilles tendon. I discussed the appropriate radiological procedure with a radiologist and they recommend initially an ultrasound of the Achilles tendon and if that was unhelpful then an MRI scan.
The ultrasound was completely normal! There was no evidence of an Achilles tendon tear!
A further diagnostic procedure was requested. Continue to the next question.
-
Question 20 of 54
20. Question
Category: Case 13The ultrasound was completely normal! There was no evidence of an Achilles tendon tear!
What would you do next?
Correct
The correct answer is Start Again!
A reasonable answer would have been refer for another opinion. However, that is not the point we are trying to make here.
Shortly after starting my own private practice I saw a patient where, at the end of the history and examination, I did not have any idea what was wrong or what to do next. I asked my mentor Dr Arthur Schwieger who was in his rooms on the other side of the corridor. He said two words that have echoed in my ears ever since “start again”
I asked the patient to return, repeated the examination, confirmed the findings. I had allowed my presumptive diagnosis to influence my interpretation of the tests. It is clear that these two bedside tests are not specific for a ruptured Achilles tendon. I then performed electromyography and H reflex. There was evidence of denervation in the gastrocnemius muscles and the H reflex was delayed on the symptomatic side confirming an S1 radiculopathy. This diagnosis should have been obvious to me from the start, I allowed myself to be influenced by two bedside tests that I found on the Internet to diagnose my suspected diagnosis of a ruptured Achilles tendon. I am unable to find any papers that document the sensitivity and specificity of Thompson’s test, but like all tests it will have a less than 100% sensitivity and specificity.
Why would an Achilles tendon rupture in the middle of the night when someone is lying in bed, it just wouldn’t happen. I jumped to conclusions without considering the facts carefully enough.
Ultrasound is the investigation of choice for a suspected Achilles tendon, if it is normal an MRI scan will not help. I have appended to the end of this case a detailed discussion of MRI versus ultrasound for musculoskeletal medicine that is an excellent reference source.
One should never hesitate to refer the patient for a second opinion, particularly if they request one, never refuse.
If you tell the patient you can’t find anything wrong they will simply go to another doctor.
Incorrect
The correct answer is Start Again!
A reasonable answer would have been refer for another opinion. However, that is not the point we are trying to make here.
Shortly after starting my own private practice I saw a patient where, at the end of the history and examination, I did not have any idea what was wrong or what to do next. I asked my mentor Dr Arthur Schwieger who was in his rooms on the other side of the corridor. He said two words that have echoed in my ears ever since “start again”
I asked the patient to return, repeated the examination, confirmed the findings. I had allowed my presumptive diagnosis to influence my interpretation of the tests. It is clear that these two bedside tests are not specific for a ruptured Achilles tendon. I then performed electromyography and H reflex. There was evidence of denervation in the gastrocnemius muscles and the H reflex was delayed on the symptomatic side confirming an S1 radiculopathy. This diagnosis should have been obvious to me from the start, I allowed myself to be influenced by two bedside tests that I found on the Internet to diagnose my suspected diagnosis of a ruptured Achilles tendon. I am unable to find any papers that document the sensitivity and specificity of Thompson’s test, but like all tests it will have a less than 100% sensitivity and specificity.
Why would an Achilles tendon rupture in the middle of the night when someone is lying in bed, it just wouldn’t happen. I jumped to conclusions without considering the facts carefully enough.
Ultrasound is the investigation of choice for a suspected Achilles tendon, if it is normal an MRI scan will not help. I have appended to the end of this case a detailed discussion of MRI versus ultrasound for musculoskeletal medicine that is an excellent reference source.
One should never hesitate to refer the patient for a second opinion, particularly if they request one, never refuse.
If you tell the patient you can’t find anything wrong they will simply go to another doctor.
-
Question 21 of 54
21. Question
Category: Case 14A 42-year-old teacher bent down to unplug his laptop computer from the power point beneath the desk. He experienced the sudden onset of a brief sharp pain in his lower neck radiating over his right shoulder and down to his right index finger. The pain increased in severity over the next hour and was associated with electric shock like pains down the right arm to the index finger. This pain was associated with altered sensation in the right index finger on both the palm and dorsal aspect.
He consulted his general practitioner who thought that he had injured his shoulder and recommended a period of rest and time off work.
An ultrasound of the shoulder confirmed bursitis.
However, his symptoms persisted following a steroid injection into the bursa.
What do you think the most likely diagnosis is?
Correct
Explanation:
The distribution of this man’s pain was in the area supplied by the C6 nerve root and the associated sensory symptoms clearly indicate that his pain was neurological in origin. A bursa of his shoulder cannot produce sensory symptoms. If the pain is related to shoulder pathology then the pain will be exacerbated by movement of the shoulder and will not be associated with any sensory symptoms. If the pain is related to nerve root compression then the pain is often but not always exacerbated by movement of the neck. Carpal tunnel syndrome can produce numbness in the second finger, but it would not produce pain radiating down from the neck to the index finger. A cervical rib can produce pain and numbness, but it compresses the lower cord of the brachial plexus and therefore the numbness would be in the fourth and fifth digits, within the distribution of the C8-T1 nerve roots.
Bursitis of the shoulder is very common in middle-aged to older people and is not always the explanation for their symptoms.
Incorrect
Explanation:
The distribution of this man’s pain was in the area supplied by the C6 nerve root and the associated sensory symptoms clearly indicate that his pain was neurological in origin. A bursa of his shoulder cannot produce sensory symptoms. If the pain is related to shoulder pathology then the pain will be exacerbated by movement of the shoulder and will not be associated with any sensory symptoms. If the pain is related to nerve root compression then the pain is often but not always exacerbated by movement of the neck. Carpal tunnel syndrome can produce numbness in the second finger, but it would not produce pain radiating down from the neck to the index finger. A cervical rib can produce pain and numbness, but it compresses the lower cord of the brachial plexus and therefore the numbness would be in the fourth and fifth digits, within the distribution of the C8-T1 nerve roots.
Bursitis of the shoulder is very common in middle-aged to older people and is not always the explanation for their symptoms.
-
Question 22 of 54
22. Question
Category: Case 15A 20 year old female has had headaches for 4 months and was referred with a suspected diagnosis of idiopathic (benign) intracranial hypertension. The headaches were present most days but not every day. They were situated in the region of the left temple, mainly constant in nature. If the headache became more severe it could be throbbing, last a week and associated with pain going down the left side of her neck posteriorly and down her back to the lower end of the shoulder blade on the left. The headache was more severe in a noisy environment, for example, at her work. On most occasions she was able to continue working. If the headache became severe, she would go home and lie down in her bedroom. There was no gastrointestinal, neurological or visual symptoms, the eyes felt sore, there was photophobia and phonophobia when the headache was severe. There was no galactorrhoea. There was no visual obscurations. She had never awoken with a headache, nor been awoken in the night by a headache. Her mother and sister both suffered from migraine.
Celebrex made her headache worse and she felt as if she had a sore swollen throat.
The patient had an Implanon implant, she worked 8-25 hours per week in a retail store and 5-10 hours per week on a farm. She read books on her iPhone and Rode horses for relaxation. Her parents felt that there might be stress because she was not happy with her job but the patient denied stress or depression. She was a non-smoker and virtually a non-drinker.
A CT scan of the brain and sinuses was reported as normal, as was her full blood examination, thyroid function tests, calcium and phosphate, iron studies, vitamin B12 and folate, CRP and ESR.
On examination the visual acuity was 6/6 bilaterally uncorrected. Her blindspot was not enlarged with testing using a 3 mm red pin. There wasn’t any swelling of the optic discs. There was marked tenderness in the left trapezius and left sternocleidomastoid muscles. The neurological examination was otherwise unremarkable.
What do you think the most likely diagnosis is?
Correct
Unilateral headache initially confined to the region of the temple would be most unlikely with idiopathic intracranial hypertension. Furthermore, there was no convincing papilloedema (there have been rare cases described of IIH without papilloedema) and the blind spot was not enlarged – it is enlarged if there is swelling of the optic disc.
Migraine rarely, if ever, causes daily headache all day every day unless the patient develops a very rare entity described as “status migraine”. We are all taught that throbbing headache occurs with migraine but it also occurs with other causes of headache and migraine is not necessarily a throbbing pain. The presence of a family history of migraine is circumstantial evidence and should not be used to make a diagnosis. Photophobia and phonophobia are also very non-specific as anyone who has ever had a hangover would be aware of. Remember the pain of migraine comes on rapidly, and a characteristic feature of common migraine, or in the new nomenclature, migraine without aura (migraine without any visual, gastrointestinal or neurological symptoms) is that patients retire to bed without a headache and wake in the middle of the night or awaken at their normal time the next morning with a severe headache.
Chronic daily headache by definition is a headache present all day every day 6 months, therefore she does not fulfil this criteria as the headaches had only been present for 4 months.
The most likely diagnosis is tension headache and there was considerable stress in her life. She was very unhappy in her work, this latter fact was elicited after one came to the conclusion that this was most likely tension headache. Tension headaches can be very difficult to treat, I usually advise people to change their lifestyle and find more time for themselves in addition to time spent at their work and with their family. The patient was advised to return if the headaches didn’t settle.
Incorrect
Unilateral headache initially confined to the region of the temple would be most unlikely with idiopathic intracranial hypertension. Furthermore, there was no convincing papilloedema (there have been rare cases described of IIH without papilloedema) and the blind spot was not enlarged – it is enlarged if there is swelling of the optic disc.
Migraine rarely, if ever, causes daily headache all day every day unless the patient develops a very rare entity described as “status migraine”. We are all taught that throbbing headache occurs with migraine but it also occurs with other causes of headache and migraine is not necessarily a throbbing pain. The presence of a family history of migraine is circumstantial evidence and should not be used to make a diagnosis. Photophobia and phonophobia are also very non-specific as anyone who has ever had a hangover would be aware of. Remember the pain of migraine comes on rapidly, and a characteristic feature of common migraine, or in the new nomenclature, migraine without aura (migraine without any visual, gastrointestinal or neurological symptoms) is that patients retire to bed without a headache and wake in the middle of the night or awaken at their normal time the next morning with a severe headache.
Chronic daily headache by definition is a headache present all day every day 6 months, therefore she does not fulfil this criteria as the headaches had only been present for 4 months.
The most likely diagnosis is tension headache and there was considerable stress in her life. She was very unhappy in her work, this latter fact was elicited after one came to the conclusion that this was most likely tension headache. Tension headaches can be very difficult to treat, I usually advise people to change their lifestyle and find more time for themselves in addition to time spent at their work and with their family. The patient was advised to return if the headaches didn’t settle.
-
Question 23 of 54
23. Question
Category: Case 16A 61-year-old man had noticed that his left eyelid was coming down over his left eye some three months ago, it was worse particularly at the end of the day. He felt the occasional muscle twitch beneath the right eyebrow but no definite drooping of the right eyelid. He denied any diplopia, he had no symptoms referable to his bulbar musculature and he was not aware of any limb weakness when specifically asked.
He had a past history of benign prostatic hypertrophy for which she took duodart, haemochromatosis undergoing venesection. He had also had a carcinoma of the bowel, it required a colostomy and the colostomy was not revised for many many months, the site of the colostomy was complicated by the development of a small hernia.
He was married, his wife and two children were well, he had three grandchildren one who has autism. He is a non-smoker drinks to 3 glasses of wine per day and worked in a rural store. His brother has haemochromatosis.
On examination I was unable to detect any fatigable ptosis, there was no impairment of ocular movements, there was no neck flexion or neck extension weakness and there was no facial weakness.. The left biceps and brachioradialis reflexes were absent all other reflexes are preserved. I was unable to detect any lymphadenopathy.
What is your diagnosis?
Correct
The correct diagnosis proved to be myasthenia gravis.
Incorrect
The correct diagnosis proved to be myasthenia gravis.
-
Question 24 of 54
24. Question
Category: Case 16Diagnostic tests were performed what were they?
Correct
All tests are used in the diagnosis of Myasthenia Gravis. Repetitive exercise of his limbs resulted in severe weakness that recovered with 30 seconds, only to recur with repeat testing.
Repetitive stimulation studies may demonstrate a decremental and it is usually done on a proximal muscle such as the trapezius.
A Tensilon test looks for the reversal of weakness (initially a 2mg dose and then after a pause 8mg as some are sensitive to small doses others need a bigger dose) looks for resolution of weakness with onset within 30 seconds of injection and reappearance of weakness 2-3 minutes after the injection. Atropine is given first to avoid vomiting induced by then Tensilon. This test in not particularly easy if there is no fixed deficit, eg permanent ptosis.
SFEMG (single fibre EMG) will detect increase jitter in myasthenia but it is not specific for myasthenia, it can occur in motor neuron disease and Lambert-Eaton Syndrome.
Thyroid function tests are indicated as it is not uncommon to have co-existent thyroid disease.
The Ice Test is very useful if th4ere is ptosis. A block of ice is applied to the drooping eyelid and it magically opens!
Incorrect
All tests are used in the diagnosis of Myasthenia Gravis. Repetitive exercise of his limbs resulted in severe weakness that recovered with 30 seconds, only to recur with repeat testing.
Repetitive stimulation studies may demonstrate a decremental and it is usually done on a proximal muscle such as the trapezius.
A Tensilon test looks for the reversal of weakness (initially a 2mg dose and then after a pause 8mg as some are sensitive to small doses others need a bigger dose) looks for resolution of weakness with onset within 30 seconds of injection and reappearance of weakness 2-3 minutes after the injection. Atropine is given first to avoid vomiting induced by then Tensilon. This test in not particularly easy if there is no fixed deficit, eg permanent ptosis.
SFEMG (single fibre EMG) will detect increase jitter in myasthenia but it is not specific for myasthenia, it can occur in motor neuron disease and Lambert-Eaton Syndrome.
Thyroid function tests are indicated as it is not uncommon to have co-existent thyroid disease.
The Ice Test is very useful if th4ere is ptosis. A block of ice is applied to the drooping eyelid and it magically opens!
-
Question 25 of 54
25. Question
Category: Case 17A 50 yo woman awoke feeling well, but later that morning prior to going to work she began to feel vaguely unwell. She says she can’t remember most of the day and found herself on the floor on two occasions and slumped over her desk on another occasion. She was unable to remember people’s names nor what she did for a living. She had called a client, and he established that her speech was slurred but he did not notice anything else. Her symptoms resolved within six hours. An MRI scan and EEG were normal.
What would you do to sort his out?
Correct
The film ET has a classic line “Phone home”. I have often been able to make many a diagnosis by obtaining a blow by blow description from an eye-witness.
The first “diagnostic procedure” was to call an eyewitness. The episode had occurred whilst the patient was doing yoga, at the stage where she was told to take deep breaths for relaxation. The patient stated that she needed to go to the toilet but when she stood up she actually sat in a chair and her limbs started trembling and jerking while she was fully conscious. This episode lasted 15-20 seconds and she seemed to pass out for 20 or 30 seconds with her eyes closed and being motionless. There was incontinence of urine. On coming to, she was fully oriented, not confused, although did not understand what had happened.
Incorrect
The film ET has a classic line “Phone home”. I have often been able to make many a diagnosis by obtaining a blow by blow description from an eye-witness.
The first “diagnostic procedure” was to call an eyewitness. The episode had occurred whilst the patient was doing yoga, at the stage where she was told to take deep breaths for relaxation. The patient stated that she needed to go to the toilet but when she stood up she actually sat in a chair and her limbs started trembling and jerking while she was fully conscious. This episode lasted 15-20 seconds and she seemed to pass out for 20 or 30 seconds with her eyes closed and being motionless. There was incontinence of urine. On coming to, she was fully oriented, not confused, although did not understand what had happened.
-
Question 26 of 54
26. Question
Category: Case 17For the same woman, there was a previous episode many years beforehand. At that time she felt as if there was a pressure suit all over her body, that she froze whilst walking to the bathroom with her next recollection being sitting in a chair where she apparently had convulsed and had been incontinent of urine. The patient, however, said that following this episode she was not confused.
In recent years she had experienced episodes of lightheadedness lasting anything from minutes to hours, fluctuating in severity when present for prolonged periods of time. There was a feeling of trembling inside, dryness of the mouth, difficulty getting her breath and tingling in the right arm.
What is your diagnosis?
Correct
The correct answer is hyperventilation syndrome. Only tonic-clonic and atonic seizures make the patient fall. An tonic-clonic seizure has little or no warning, the patient is “out to it” and they are confused after the episode. AN atonic seizure has no warning the patient drops but remains fully conscious and is not confused afterwards.
A Stokes-Adams attack is complete heart-block. There is no warning, a fall with loss of conscious and no confusion after the episode unless they hit their head in the fall
When symptoms vary in nature and duration between episodes this is usually a clue that the problem is of psychological origin. Thus an non-epileptic seizure is a possibility
It is the long warning and the dryness of the mouth, difficulty getting her breath and tingling in the right arm that are the clues to the correct diagnosis of hyperventilation syndrome. Her incontinence probably reflected a full bladder. It has to be said that it is unusual for the patient to fall with hyperventilation syndrome usually they remain conscious and can describe the entire episode. Her symptoms including “out to it” were reproduced by vigorous hyperventilatoin
Incorrect
The correct answer is hyperventilation syndrome. Only tonic-clonic and atonic seizures make the patient fall. An tonic-clonic seizure has little or no warning, the patient is “out to it” and they are confused after the episode. AN atonic seizure has no warning the patient drops but remains fully conscious and is not confused afterwards.
A Stokes-Adams attack is complete heart-block. There is no warning, a fall with loss of conscious and no confusion after the episode unless they hit their head in the fall
When symptoms vary in nature and duration between episodes this is usually a clue that the problem is of psychological origin. Thus an non-epileptic seizure is a possibility
It is the long warning and the dryness of the mouth, difficulty getting her breath and tingling in the right arm that are the clues to the correct diagnosis of hyperventilation syndrome. Her incontinence probably reflected a full bladder. It has to be said that it is unusual for the patient to fall with hyperventilation syndrome usually they remain conscious and can describe the entire episode. Her symptoms including “out to it” were reproduced by vigorous hyperventilatoin
-
Question 27 of 54
27. Question
Category: Case 18A 52 year old woman 4 months before being seen became aware of a sensitive area on the top of her right foot between the first and second toes, this was sensitive when she wore shoes and sensitive when the sheet touches that area. This sensitivity was replaced by numbness in the same distribution and over the last three months she had noticed a shooting pain between the first and second digits that came on without any obvious reason at all. Last night her daughter pushed distally between the first and second toes near the base of the toes and elicited pain.
The examination revealed normal power and reflexes. There was altered pain sensation affecting the lateral aspect of the first digit in the medial aspect of the second digit spreading proximally from the tip of the first digit to a distance of 15.5 cm up the foot between the first and second metatarsal bones. There was tenderness on palpation of the metatarsophalangeal joints of the 2nd, 3rd, and 4th digits. Palpation 6.5 cm distal to the line connecting the medial and lateral malleoli elicited the shooting pain.
What is your diagnosis?
Correct
Osteoarthritis can produce pain affecting the metatarso-phalangeal joints but will not be associated with sensory loss.
The distribution of the sensory findings is that of the deep peroneal not the superficial peroneal nerve but this cannot explain the pain on palpation of the 2nd, 3rd and 4th metatarso-phalangeal joints
Morten’s neuroma specifically refers to a neuroma on the nerve between the third and fourth digits.
Thus, the information given to you so far means you cannot be certain about the diagnosis.
Beneath the site that elicited the shooting pain was a palpable lump and this patient almost certainly has a neuroma on the deep peroneal nerve, but this cannot explain the pain involving the metatarso-phalangeal joints.
The MRI scan of her foot showed inter-metatarsal bursal fluid at the 1st, 2nd and 3rd interspaces, this usually occurs in mechanical stress. This is almost certain an explanation for the pain elicited by her daughter and the pain elicited when we examine those joints.
Although we are taught Occam’s razor, it is not always correct and one needs to carefully analyse every aspect of the patient’s symptoms and signs to come to the correct diagnosis in this patient.
Incorrect
Osteoarthritis can produce pain affecting the metatarso-phalangeal joints but will not be associated with sensory loss.
The distribution of the sensory findings is that of the deep peroneal not the superficial peroneal nerve but this cannot explain the pain on palpation of the 2nd, 3rd and 4th metatarso-phalangeal joints
Morten’s neuroma specifically refers to a neuroma on the nerve between the third and fourth digits.
Thus, the information given to you so far means you cannot be certain about the diagnosis.
Beneath the site that elicited the shooting pain was a palpable lump and this patient almost certainly has a neuroma on the deep peroneal nerve, but this cannot explain the pain involving the metatarso-phalangeal joints.
The MRI scan of her foot showed inter-metatarsal bursal fluid at the 1st, 2nd and 3rd interspaces, this usually occurs in mechanical stress. This is almost certain an explanation for the pain elicited by her daughter and the pain elicited when we examine those joints.
Although we are taught Occam’s razor, it is not always correct and one needs to carefully analyse every aspect of the patient’s symptoms and signs to come to the correct diagnosis in this patient.
-
Question 28 of 54
28. Question
Category: Case 19A 56-year-old man presented with long-standing stiffness in his neck and intermittent pins and needles affecting the third and fourth digits of both hands. On the right side he had a burning discomfort that could awaken him from his sleep. The symptoms appeared to improve when he moved his neck and his hands together.
A CT scan of the cervical spine demonstrated bilateral foraminal stenoses at C5-6.
What would you do next?
Correct
Please continue to the next question for the diagnosis.
Incorrect
Please continue to the next question for the diagnosis.
-
Question 29 of 54
29. Question
Category: Case 19What is your diagnosis for this 56 yo man?
Correct
Most 56-year-old patients will have degenerative disease of the cervical spine. The distribution of the sensory symptoms is not reflective of that of the C6 nerve root and therefore the findings on the CT scan of the cervical spine are irrelevant or an incidental finding. Disc disease compresses the nerve below the vertebrae ie C6 nerve below C5 vertebra, thus it cannot be a C5 radiculopathy.
An MRI scan is more sensitive at detecting nerve root compression in the cervical and thoracic spine while a CT scan is adequate in most cases with problems in the lumbosacral spine.
He was booked for cervical surgery after an MRI scan of the spine.
The patient himself requested another opinion for another diagnosis. The examination at that time demonstrated weakness of both APB muscles and also sensory loss affecting the tips of the digits of the lateral 4 1/2 fingers of both hands on the palm up to the wrist but not the dorsal aspect.
Two provocative tests were performed at the bedside, Tinel’s sign where the median nerve is precast using attendant hammer commencing 6 cm above the wrist and coming downwards and into the proximal palm, a positive test is when the patient gets tingling or numbness in the hand and fingers, not pain at the site of percussion. Phalen’s sign is where the wrists are either forcibly flexed or extended and once again a positive test is where the patient experiences tingling or numbness in the fingers, pain is not a positive sign. These tests don’t have a high sensitivity and specificity.
The diagnostic procedure of course was nerve conduction studies that confirmed the patients diagnosis i.e. bilateral carpal tunnel syndrome.
The lateral aspect of this case highlights the importance of establishing the exact nature and, more importantly, the distribution of the symptoms. If he had C6 nerve root compression he would more likely have pain radiating down his arm, although you can have painless nerve root compression. If the compression affected the dorsal root then you would expect to see sensory loss in the C6 dermatome which is the lateral aspect of the forearm and the lateral two digits, not the third and fourth digits. If this were an ulnar nerve lesion you would not expect the third digit to be affected.
If the motor nerve root was affected one would expect to see weakness affecting the C6 nerve root which would give you partial weakness of biceps, brachioradialis and to a lesser extent triceps, there may also be some weakness of supraspinatus and infraspinatus and deltoid, together with a reduced biceps reflexes possibly an absent brachioradialis reflex.
Bilateral symptoms of nerve root compression is very rare indeed, if a patient has bilateral numbness in the hands it is far more likely to be related to carpal tunnel syndrome because this is the most common condition affecting the nerves in the upper limbs.
Incorrect
Most 56-year-old patients will have degenerative disease of the cervical spine. The distribution of the sensory symptoms is not reflective of that of the C6 nerve root and therefore the findings on the CT scan of the cervical spine are irrelevant or an incidental finding. Disc disease compresses the nerve below the vertebrae ie C6 nerve below C5 vertebra, thus it cannot be a C5 radiculopathy.
An MRI scan is more sensitive at detecting nerve root compression in the cervical and thoracic spine while a CT scan is adequate in most cases with problems in the lumbosacral spine.
He was booked for cervical surgery after an MRI scan of the spine.
The patient himself requested another opinion for another diagnosis. The examination at that time demonstrated weakness of both APB muscles and also sensory loss affecting the tips of the digits of the lateral 4 1/2 fingers of both hands on the palm up to the wrist but not the dorsal aspect.
Two provocative tests were performed at the bedside, Tinel’s sign where the median nerve is precast using attendant hammer commencing 6 cm above the wrist and coming downwards and into the proximal palm, a positive test is when the patient gets tingling or numbness in the hand and fingers, not pain at the site of percussion. Phalen’s sign is where the wrists are either forcibly flexed or extended and once again a positive test is where the patient experiences tingling or numbness in the fingers, pain is not a positive sign. These tests don’t have a high sensitivity and specificity.
The diagnostic procedure of course was nerve conduction studies that confirmed the patients diagnosis i.e. bilateral carpal tunnel syndrome.
The lateral aspect of this case highlights the importance of establishing the exact nature and, more importantly, the distribution of the symptoms. If he had C6 nerve root compression he would more likely have pain radiating down his arm, although you can have painless nerve root compression. If the compression affected the dorsal root then you would expect to see sensory loss in the C6 dermatome which is the lateral aspect of the forearm and the lateral two digits, not the third and fourth digits. If this were an ulnar nerve lesion you would not expect the third digit to be affected.
If the motor nerve root was affected one would expect to see weakness affecting the C6 nerve root which would give you partial weakness of biceps, brachioradialis and to a lesser extent triceps, there may also be some weakness of supraspinatus and infraspinatus and deltoid, together with a reduced biceps reflexes possibly an absent brachioradialis reflex.
Bilateral symptoms of nerve root compression is very rare indeed, if a patient has bilateral numbness in the hands it is far more likely to be related to carpal tunnel syndrome because this is the most common condition affecting the nerves in the upper limbs.
-
Question 30 of 54
30. Question
Category: Case 20A 36-year-old man presents with nine months of increasing hoarseness of his voice, generalised weakness and loss of muscle bulk. The hoarseness was better first thing in morning but there was no obvious fluctuation throughout the day.
Just prior to the onset he had presented with confusion, altered and aggressive behaviour. He had abnormal liver function tests and a CT scan confirmed severe cirrhosis of the liver with portal hypertension and oesophageal varices. Since the age of 17 he had drunk two slabs (48 cans-96 standard drinks) of alcohol every weekend and since the age of 30, 12 standard drinks per day. His CT scan showed abnormalities in the head of the caudate nucleus consistent with hepatic encephalopathy. The CE 282Y mutation was detected as a heterozygote for haemachromatosis, his HFE Gene mutation was not detected. His serum copper was normal his ceuruloplasmin was mildly reduced to 0.13 (normal 0.15-0.30). The alpha-fetoprotein was raised at 13 (normal <8), his ANF, anti-mitochondrial and smooth muscle antibodies were negative. Iron studies were normal. His PT INR was prolonged at 1.6. His serum testosterone was found to be low. Initially he was treated with testosterone without benefit, when he presented with confusion, he was treated for hepatic encephalopathy and although his altered and aggressive behaviour together with his confusion cleared the hoarseness of his voice worsened.
When examined his voice was so hoarse it was inaudible. Despite his subjective symptoms of weakness there was no objective neurological signs. There was no fatigable weakness with repetitive exercise and no ptosis or impaired ocular movements.
What do you think the cause of his cirrhosis of the liver was due to?
Correct
A diagnosis of alcoholic hepatitis was made (Dr jump to conclusions) based on his excessive alcohol consumption (circumstantial evidence) and he was treated appropriately, however he lost his ability to communicate due to severe hoarseness. His tests for haemochromatosis showed he was heterozygote, making this diagnosis an unlikely.casue for his cirrhosis His autoantibody tests for autoimmune cirrhosis. Anti-mitochondrial antibodies (AMA) are present in sera of approximately 90-95% of patients with primary biliary cirrhosis (PBC). Smooth muscle antibodies (SMA) of IgG class are found in 92% of sera from patients with chronic active hepatitis but in only 20% of sera from primary biliary cirrhosis, whereas the incidence of IgM-SMA is higher in PBC (67%) than in chronic active hepatitis (38%).
Myasthenia can give a hoarse voice extremely rare isolated laryngeal myasthenia cases have been described but it would be very unlikely to occur . Myasthenia does not cause cirrhosis.
The low ceruloplasmin raised the possibility of Wilson’s disease but it was borderline low and this can occur with liver disease . A normal serum copper does not exclude this entity.
A search for hoarseness with chronic liver disease did not find any cases but a search for hoarseness with Wilson’s disease found several case reports. The dysphonia may be related to dystonic vocal cords.
A slit lamp examination detected Kayser-Fleischer rings and his 24 hour urinary copper was markedly elevated confirming the diagnosis of Wilson’s disease.
The dysphonia was treated with Botox injections.
Incorrect
A diagnosis of alcoholic hepatitis was made (Dr jump to conclusions) based on his excessive alcohol consumption (circumstantial evidence) and he was treated appropriately, however he lost his ability to communicate due to severe hoarseness. His tests for haemochromatosis showed he was heterozygote, making this diagnosis an unlikely.casue for his cirrhosis His autoantibody tests for autoimmune cirrhosis. Anti-mitochondrial antibodies (AMA) are present in sera of approximately 90-95% of patients with primary biliary cirrhosis (PBC). Smooth muscle antibodies (SMA) of IgG class are found in 92% of sera from patients with chronic active hepatitis but in only 20% of sera from primary biliary cirrhosis, whereas the incidence of IgM-SMA is higher in PBC (67%) than in chronic active hepatitis (38%).
Myasthenia can give a hoarse voice extremely rare isolated laryngeal myasthenia cases have been described but it would be very unlikely to occur . Myasthenia does not cause cirrhosis.
The low ceruloplasmin raised the possibility of Wilson’s disease but it was borderline low and this can occur with liver disease . A normal serum copper does not exclude this entity.
A search for hoarseness with chronic liver disease did not find any cases but a search for hoarseness with Wilson’s disease found several case reports. The dysphonia may be related to dystonic vocal cords.
A slit lamp examination detected Kayser-Fleischer rings and his 24 hour urinary copper was markedly elevated confirming the diagnosis of Wilson’s disease.
The dysphonia was treated with Botox injections.
-
Question 31 of 54
31. Question
Category: Case 21A 49 year old man noticed 8 months before being seen patches of warmth on his right thigh but only whilst he was playing badminton. Over the ensuring month however a numbness spread from his right thigh down his right leg to his foot, he thinks the whole leg but is not sure whether it encompassed the whole foot. The numbness also spread up the right side of his trunk to beneath his armpit. No pain and no sphincter disturbance. PH of low back pain and non-hodgkins lymphoma 1o years prior.
The examination revealed increased tone in the left arm with an obvious supinator catch, all reflexes were brisk, there were positive finger jerks bilaterally. There was no sensory loss in the upper limbs. The abdominal reflexes were preserved. There was altered pain and temperature sensation affecting the right leg the right side of the trunk and up to the level of T2 and to the midline. Vibration and proprioception were preserved. The strength in the upper limbs was normal, in the left leg however there was increased tone with sustained ankle clonus, and extensor left plantar and mild weakness of hip flexion knee flexion and dorsiflexion of the left foot. The cranial nerves were normal.
Where is the lesion?
Correct
He has a classic Brown-Sequard syndrome indicating a lesion in one side of the spinal cord. The motor pathways cross at the level of the foramen magnum, thus if he has in the left leg the problem must be on the left side of the spinal. This fists with the altered sensation affecting the right side of the body as the spinothalamic pathway crosses the moment it enters the spinal cord, thus the spinothalamic pathway is also affected on the left side of the spinal cord. The upper level of the sensory signs is the armpit about T2. The upper motor neuron signs in the left arm indicate the lesion was above C5 on the left.
Incorrect
He has a classic Brown-Sequard syndrome indicating a lesion in one side of the spinal cord. The motor pathways cross at the level of the foramen magnum, thus if he has in the left leg the problem must be on the left side of the spinal. This fists with the altered sensation affecting the right side of the body as the spinothalamic pathway crosses the moment it enters the spinal cord, thus the spinothalamic pathway is also affected on the left side of the spinal cord. The upper level of the sensory signs is the armpit about T2. The upper motor neuron signs in the left arm indicate the lesion was above C5 on the left.
-
Question 32 of 54
32. Question
Category: Mri vs UltrasoundMRI vs Ultrasound
Posted on August 18, 2015, updated on August 24, 2015 by London Imaging Centre
Dr Qaiser MalikDo I refer for MRI or Ultrasound?
Consultant Radiologist
London Imaging CentreThis is a debate that ‘tos and fros’. When it comes to musculoskeletal imaging both modalities can be used to assess most joints and conditions, but using the appropriate modality for the correct indication can make the diagnosis more accurate as well as saving the patient numerous investigations.
This blog will attempt to demystify some of the common misconceptions.
Firstly most people feel that an MRI scan must somehow be superior to an Ultrasound scan because it is usually more expensive and the images obtained seem to be more elaborate. Referring clinicians can also have a go at looking at MRI images and importantly show patients at the time of consultation. All of this makes MRI more tempting when it comes to MSK ailments.
However, it must be noted that USS has considerable advantages over MRI:
In general terms it is quicker and more easily accessible, especially as more and more sonographers and radiologists subspecialise in MSK USS. In the right hands USS can be an invaluable tool. USS does not have the claustrophobic issues that come with MRI. It is cheaper, a factor that is an important consideration in the current financial climate faced by both the NHS and the private sector. USS also allows dynamic scanning with direct interaction between the operator and the patient, as well as allowing guided intervention to be performed at the same time. Here at London Imaging Centre all our ultrasound scans are performed by Consultant Radiologists.
So, when should we use MRI and when do we use USS?
If we take each MSK region in turn we can highlight conditions and situations when each one is more appropriate than the other.
Starting with the Spine – USS has no role in the assessment of spinal conditions and MRI is the investigation of choice as it is vastly superior for imaging of the spine. MRI can exquisitely demonstrate disc pathology, vertebral anatomy and cord abnormalities.
Moving to the Shoulder, if the clinical history is one of impingement and potential rotator cuff pathology then USS is preferred, USS can dynamically assess the rotator cuff and subacromial subdeltoid bursa and demonstrate active impingement. USS can then be used for an injection if clinically indicated or requested.
MRI is perfect for assessing intra-articular pathologies such as labral abnormalities which would result in instability.
USS guided MR arthrograms can increase the sensitivity and specificity of labral pathologies.
With regards to the Elbow and Wrist – if a tendinous pathology is suspected such as lateral epicondylitis (tennis elbow) or De Quervain’s Tenosynovitis – then go for USS. This will allow dynamic scanning with ability to add Doppler to demonstrate inflammation and also intervene with an injection if appropriate. However if a bony or ligamentous injury is suspected then MRI is far superior. So for scaphoid fractures and TFCC injuries MRI is the investigation of choice.With the Hand, a combination of USS and MRI is used depending on local expertise to assess and look for tendon abnormalities such as pulley tears, or to look for synovitis.
Moving to the lower limbs, at the Hip joint – if superficial hip structures need to be assessed such as the greater trochanter, gluteal tendons, iliopsoas or hamstrings then USS is very good for the same reasons as above. Dynamic scanning of the hip is particularly useful when assessing the snapping hip syndrome.
For internal hip issues, such as labral pathologies or CAM deformities and impingement then MRI or MR Arthrogram is preferred. MRI is usually also preferred for groin pathologies such as adductor sprains and sportsman’s hernia syndromes; although it must be said USS in the right hands can also give the diagnosis accurately and also guide intervention.
With the Knee USS is very limited and only really useful for assessing the posterior fossa, the quadriceps and patella tendons.
MRI is ideal for assessing all aspects of the knee and it is rare for a patient to have an USS of the knee. MRI can easily identify meniscal pathologies, ligamentous and tendinous injuries, as well as showing up characteristic bony oedema patterns associated with injuries.
The Foot and Ankle also has specific indications for USS and others for MRI. If the clinical suspicion is of Achilles’ tendon or plantar fascia pathologies then USS is far superior to MRI, this is particularly the case if Achilles’ tendon rupture is suspected. In this case a dynamic scan is invaluable for the clinician to decide management. With both Achilles and plantar fascia diagnoses intervention must be done under USS to ensure safe distribution of the injectant – an incorrectly placed injection can lead to rupture and catastrophic results.
The flexor, extensor and peroneal tendons can be assessed using both MRI and USS and often with these complex patients they end up having both scans. MRI is gold standard for stress responses and stress fractures. MRI is also used to assess complex injuries such as the Lisfranc injury which can be unstable and require surgical intervention. Mortons neuromas can be assessed using both USS and MRI.
Finally with regards to lumps and bumps it is widely accepted that USS should be the first line choice followed by MRI, if the diagnosis is either in doubt or sinister in nature.
In conclusion MRI and USS have their specific strengths and weaknesses, understanding the uses of each modality appropriately can result in a much better, more accurate diagnosis with possible treatment and ultimately the right test for the right patient at the right time.
Dr Malik is a consultant radiologist here at London Imaging Centre, specialising in musculoskeletal radiology. He also performs private MSK and general ultrasound here on a Wednesday or Thursday. To refer a patient, download our referral form here. If you would like more information, or have a query regarding a referral, our team would be happy to help. Please get in touch by calling 020 7467 8800 or email help@london-imaging.co.uk.-
No answer a good review article
Correct
No question just a good review
Incorrect
No question just a good review
-
-
Question 33 of 54
33. Question
Category: Case 22A 42-year-old man presents with three years of intermittent headache. These headaches were bilateral, constant, associated with nausea and vomiting but no other symptoms. They would come once every two months and last anything from one week to 2 months.
He then presented with a three day history of of horizontal double vision on looking to the left and progressive weakness in all 4 limbs in the absence of any sensory symptoms, sphincter disturbance or symptoms referable to his other cranial nerves.
Examination revealed a left 6th nerve palsy, the cranial nerves were otherwise normal. He had weakness in all four limbs but was patchy and appeared to be in an upper motor neuron pattern, his reflexes were present but not brisk, the tone was normal, both planners were downgoing and there was no sensory deficit.
Correct
The correct answer is D). I made an incorrect diagnosis when I first saw this man, I allowed his past history to influence my thinking. I thought he might have had something giving him intermittent ventricular obstruction such as a colloid cyst or a lesion in the fourth ventricle causing the intermittent headaches and then subsequently causing what appeared to be a pontine syndrome with involvement of the left sixth nerve in all four limbs causing weakness. I ordered an MRI scan of the brain that was completely normal! The error was assuming that the headaches must be involved in the current illness, but he didn’t have the headache at the time he developed the weakness and if the diagnosis that I had originally made was correct he should have had the headache. In fact over the next 24 hours he developed increasing weakness of his limbs that had spread to involve all muscle groups with loss of reflexes except for the ankle reflexes! There was neck flexion weakness. The correct diagnosis was acute inflammatory demyelinating peripheral neuropathy (AIDP). Botulinum poisoning s can resemble AIDP, the clue “pentad” of symptoms signs with nausea and vomiting, dysphagia, diplopia, dilated and fixed pupils, dry mouth and throat. I would have caution, sometimes these patients have been reported as responding to phoneme the drug used to test for myasthenia gravis. Myasthenia gravis can cause diplopia and for limb weakness, the reflexes would be normal. Other conditions to consider would be Lambert-Eaton syndrome and tickborne paralysis.
Incorrect
The correct answer is D). I made an incorrect diagnosis when I first saw this man, I allowed his past history to influence my thinking. I thought he might have had something giving him intermittent ventricular obstruction such as a colloid cyst or a lesion in the fourth ventricle causing the intermittent headaches and then subsequently causing what appeared to be a pontine syndrome with involvement of the left sixth nerve in all four limbs causing weakness. I ordered an MRI scan of the brain that was completely normal! The error was assuming that the headaches must be involved in the current illness, but he didn’t have the headache at the time he developed the weakness and if the diagnosis that I had originally made was correct he should have had the headache. In fact over the next 24 hours he developed increasing weakness of his limbs that had spread to involve all muscle groups with loss of reflexes except for the ankle reflexes! There was neck flexion weakness. The correct diagnosis was acute inflammatory demyelinating peripheral neuropathy (AIDP). Botulinum poisoning s can resemble AIDP, the clue “pentad” of symptoms signs with nausea and vomiting, dysphagia, diplopia, dilated and fixed pupils, dry mouth and throat. I would have caution, sometimes these patients have been reported as responding to phoneme the drug used to test for myasthenia gravis. Myasthenia gravis can cause diplopia and for limb weakness, the reflexes would be normal. Other conditions to consider would be Lambert-Eaton syndrome and tickborne paralysis.
-
Question 34 of 54
34. Question
Category: Case 23A 62-year-old woman presents with pain affecting the right side of her face arm leg and trunk. It had developed suddenly some weeks ago. There is a past history of diabetes. The examination reveals impaired pain and temperature sensation splitting the midline, affecting her right face, head, arm, trunk and leg.
Where in the nervous system is her problem?
Correct
The altered sensation to pain and temperature indicates that the spinothalamic tract is affected on the left side, using the sensory signs alone all you can say is that there is a lesion on the left side of the brain above the mid-pons where the 5th cranial for trigeminal nerve is located. This is your “Meridian of longitude”. The “parallel of latitude” is the presence of the pain in the same distribution. This is the very rare central pain syndrome seen in thalamic infarct. In 99.9 % of cases when pain is of neurological origin it clearly points to a problem in the peripheral nervous system.
Incorrect
The altered sensation to pain and temperature indicates that the spinothalamic tract is affected on the left side, using the sensory signs alone all you can say is that there is a lesion on the left side of the brain above the mid-pons where the 5th cranial for trigeminal nerve is located. This is your “Meridian of longitude”. The “parallel of latitude” is the presence of the pain in the same distribution. This is the very rare central pain syndrome seen in thalamic infarct. In 99.9 % of cases when pain is of neurological origin it clearly points to a problem in the peripheral nervous system.
-
Question 35 of 54
35. Question
Category: Case 24A 65-year-old woman presents with 5 years of tonic-clonic seizures and progressive decline in cognitive function. She is eventually referred to a University Hospital when she developed weakness in both legs and the right arm. She has bilateral grasp reflexes and a left palmo-mental reflexes. The tone is increased in the upper limbs, the lower limbs are flaccid and paraplegic.
What test would you do to establish the diagnosis?
Correct
Weakness in both legs in the right arm almost certainly reflects involvement of the central nervous system, it would be very atypical although not entirely impossible to have a peripheral nervous system problem where three limbs are affected. The increased tone points to a central nervous system problem, flaccid paraplegia can occur in central nervous system problems. The presence of seizures and impaired cognitive function points to involvement of the cerebral hemispheres and the bilateral grasp reflexes and palmomental reflex point to the frontal lobe. This was a case that I saw in Rwanda, they do not have access to neuropsychology, although all this would confirm is that the problems was in the frontal lobe. They also do not have access to neuro-physiology nor do they have access to MRI scans except for two private scanners in the capital Kigali, this case was not seen in Kigali.
The correct answer was a CT scan of the brain. This demonstrated a massive meningioma affecting both frontal lobes.
Incorrect
Weakness in both legs in the right arm almost certainly reflects involvement of the central nervous system, it would be very atypical although not entirely impossible to have a peripheral nervous system problem where three limbs are affected. The increased tone points to a central nervous system problem, flaccid paraplegia can occur in central nervous system problems. The presence of seizures and impaired cognitive function points to involvement of the cerebral hemispheres and the bilateral grasp reflexes and palmomental reflex point to the frontal lobe. This was a case that I saw in Rwanda, they do not have access to neuropsychology, although all this would confirm is that the problems was in the frontal lobe. They also do not have access to neuro-physiology nor do they have access to MRI scans except for two private scanners in the capital Kigali, this case was not seen in Kigali.
The correct answer was a CT scan of the brain. This demonstrated a massive meningioma affecting both frontal lobes.
-
Question 36 of 54
36. Question
Category: Case 25A 26-year-old student presents with involuntary jerking of his upper body. It only occurs when he is seated, it does not occur when he is walking, lying in bed or asleep. There is no difficulty with his speech vision or the use of his arms or legs and there is no significant past history apart from a minor head injury where he was rendered unconscious for a short period of time many years beforehand. On examination he was able to walk without any abnormal movements, and whilst seated we asked him to elevate his legs up off the floor and this abolished the abnormal movements. The movements themselves were brief and at times lifted his buttocks up off the seat with a sudden jerk of his trunk
What is your diagnosis?
Correct
It is sometimes difficult to tell the difference between abnormal movements due to structural pathology and abnormal movements reflecting underlying psychological problems.
Incorrect
It is sometimes difficult to tell the difference between abnormal movements due to structural pathology and abnormal movements reflecting underlying psychological problems.
-
Question 37 of 54
37. Question
Category: Case 26A 66-year-old man presents with for years of painless twitching and cramps in the muscles of his legs, particularly the calves. These were not associated with exercise and did not resolve when Crestor was stopped. Seven months prior to presentation he noticed some weakness in his left leg that progressed over the next week and whilst walking he noticed a left foot drop. He had not noticed any painful muscle cramps. In the months prior to presentation he developed weakness in his right leg and some painless twitches in the muscles of the left arm. He had not lost any weight and was otherwise fit and well. Examination revealed fasciculations in the left biceps and deltoid region, mild weakness of finger extension and finger abduction and the APB bilaterally with a hand grip strength of 19 on the right and 21 on the left, tone was normal, the reflexes were brisk sensation was normal in the upper limbs. In the lower limbs there was asymmetrical weakness of hip flexion knee flexion together with dorsi flexion of both feet, the reflexes best both planters are extensive. Limbs circumference was 50 cm on the right 49 cm on the left 25 cm above the general tuberosity and 38 cm on the right, 36 on the left 12 cm below the tibial tuberosity. There were no sensory signs. He had undergone an L5 discectomy with insertion of a Titan cage for his foot drop without benefit.
Correct
Fasciculations without weakness are usually benign, this is often referred to as medical students motor neurone disease because they notice fasciculations and worry that they have that diagnosis. He has no sensory symptoms, thus anyone of the listed diagnoses are possible once weakness develops however fasciculations are more sinister. Fasciculations and weakness in upper and lower limbs can be caused by motor neurone disease, multifocal motor neuropathy or Isaac’s syndrome (Neuromyotonia). Fasciculations would indicate a lower motor neurone problem. Painful cramps with painful bulging muscles can occur with neuromyotonia and less commonly with multifocal motor neuropathy. In the latter condition the pattern of weakness should reflect involvement of multiple motor nerves.
On the information that you have been given it is not possible to differentiate between multifocal motor neuropathy, motor neurone disease for neuromyotonia.
A diagnostic procedure was performed, please proceed to the next question
Incorrect
Fasciculations without weakness are usually benign, this is often referred to as medical students motor neurone disease because they notice fasciculations and worry that they have that diagnosis. He has no sensory symptoms, thus anyone of the listed diagnoses are possible once weakness develops however fasciculations are more sinister. Fasciculations and weakness in upper and lower limbs can be caused by motor neurone disease, multifocal motor neuropathy or Isaac’s syndrome (Neuromyotonia). Fasciculations would indicate a lower motor neurone problem. Painful cramps with painful bulging muscles can occur with neuromyotonia and less commonly with multifocal motor neuropathy. In the latter condition the pattern of weakness should reflect involvement of multiple motor nerves.
On the information that you have been given it is not possible to differentiate between multifocal motor neuropathy, motor neurone disease for neuromyotonia.
A diagnostic procedure was performed, please proceed to the next question
-
Question 38 of 54
38. Question
Category: Case 26What diagnostic procedure was performed?
Correct
Incorrect
The way to differentiate between these very entities is to perform nerve conduction studies and electromyography. Wasting should not occur with myasthenia gravis and weakness is exercise induced thus this diagnosis is very unlikely. Repetitive stimulation studies are likely to be normal. The way to differentiate between multifocal motor neuropathy, neuromyotonia and motor neurone disease is using nerve conduction studies electromyography. Single fibre EMG will be abnormal in motor neurone disease and multifocal motor neuropathy, both will have increased jitter on SFEMG. The presence of conduction block on the nerve conduction studies indicate a diagnosis of multifocal motor neuropathy, the nerve conduction studies are either normal or show reduced amplitude of motor responses with motor neurone disease, both would have normal sensory conduction studies. There is no evidence of conduction block in motor neuron disease. Both would have evidence of denervation and re-innervation on electromyography.
-
Question 39 of 54
39. Question
Category: Case 27A 42 year old woman has suffered from headaches for about 18 years occurring about 3-4 times per year and lasting about 24-hour at a time. The initial symptom is a visual disturbance as if there is oil and water, it is bilateral, it doesn’t spread, and it lasts about 30 minutes. She developed photosensitivity and then a bifrontal and bioccipital constant headache associated with nausea without vomiting, there are no neurological symptoms and no phonophobia. These headaches awaken her from sleep or are present and severe first thing in morning on wakening.
In the last 2 years a second type of headache had developed. It almost invariably commences some time between 11:30 AM and 1 PM and last about two hours. It is a constant left frontal and left occipital pain 6-9/10 severity with mild photophobia, no phonophobia, photopsia, gastrointestinal or neurological symptoms. This headache increases in severity very rapidly and resolves almost as quickly. The pain may spread to the left side of the nose and it has been noticed that the left eyelid is drooping, the eye is watering slightly, she has not been aware of blockage of the nose because her nose is always blocked! Neither she or her husband have looked at the pupil during these episodes.
Treatments to date has included Topamax 25 mg twice a day that was not tolerated, propranolol up to 40 mg per day without benefit but without side-effects. She uses Relpax (Elitriptan) 80 mg when necessary for the migraine but these tend to wipe her out, she prefers to use Neurofen (Ibuprofen).
What is your diagnosis?
Correct
If you said migraine, that was the explanation for the headaches that had been occurring over the preceding 18 years. However the classic feature of cluster headache is that one can set ones watch to them, they occur at exactly the same time every day, although she is female and cluster headache is unusual in females, she had the other characteristic features which include ptosis, ipsilateral headache that was severe, watering of the eye and she almost certainly had a small pupil but hadn’t noticed it. The presence of a Horner’s Syndrome with recurrent unilateral headache is pathognomonic for cluster headache. Cluster headache can occur once per day or several times per day, it often occurs at the same time each year, bouts may last as long as six weeks although in some patients such as this one chronic cluster headache can develop.
The presence of a Horner’s Syndrome in someone presenting with a severe acute ipsilateral headache could also suggest the possibility of other pathology, in particular a carotid artery dissection.
The pain is beyond the distribution of the trigeminal nerve and therefore it cannot be trigeminal neuralgia, tension headache tends to come on towards the end of the day but it is not so stereotyped, often varying in intensity between one episode and another, fluctuating in intensity when present and not associated with ptosis or watering of the eye. Atypical facial pain presents with a more constant boring, burning discomfort in the face without such periodicity.
Incorrect
If you said migraine, that was the explanation for the headaches that had been occurring over the preceding 18 years. However the classic feature of cluster headache is that one can set ones watch to them, they occur at exactly the same time every day, although she is female and cluster headache is unusual in females, she had the other characteristic features which include ptosis, ipsilateral headache that was severe, watering of the eye and she almost certainly had a small pupil but hadn’t noticed it. The presence of a Horner’s Syndrome with recurrent unilateral headache is pathognomonic for cluster headache. Cluster headache can occur once per day or several times per day, it often occurs at the same time each year, bouts may last as long as six weeks although in some patients such as this one chronic cluster headache can develop.
The presence of a Horner’s Syndrome in someone presenting with a severe acute ipsilateral headache could also suggest the possibility of other pathology, in particular a carotid artery dissection.
The pain is beyond the distribution of the trigeminal nerve and therefore it cannot be trigeminal neuralgia, tension headache tends to come on towards the end of the day but it is not so stereotyped, often varying in intensity between one episode and another, fluctuating in intensity when present and not associated with ptosis or watering of the eye. Atypical facial pain presents with a more constant boring, burning discomfort in the face without such periodicity.
-
Question 40 of 54
40. Question
Category: Question 28A 31-year-old man presented with two months of bifrontal headache precipitated by bending over and associate with a sense of non-specific dizziness. He noticed a mild headache first thing in the morning on waking but the moment he stood up the headache was very severe, it was in the occipital to frontal region bilaterally but worse on the left. The headache was constant in nature without neck stiffness and without any visual, neurological or gastrointestinal symptoms. There was a past history and a strong family history of migraine. The neurological examination was normal. What is the most likely diagnosis?
Correct
Headaches that are precipitated upon standing and relieved by lying flat occur following a lumbar puncture and these are referred to as post-LP headache, they are due to a CSF leak. A spontaneous CSF leak can occur, often following very trivial injury to the neck or back. Patients present with the identical headache to those seen after lumbar puncture, severe on standing and abolished by lying flat with the feet slightly elevated, if the head is slightly elevated it would be greatly relieved that may not be abolished. It is generally bilateral and worse in the occipital region and frontal region.
Chronic daily headache is a headache that is essentially present all day every day, fluctuating in severity from hour to hour day to day. It can get worse with exertion but it is not abolished by lying flat. It is usually bilateral frontal and occiptial and non throbbing.
Occioital neuralgia is a unilateral stabbing or shooting pain that last seconds at a time and can recur many times per dayBenign exertional headache can be of sudden onset although often there a mild headache that increases in severity as the exertion continues. The past and family history of migraine was a red herring totally irrelevant and put here as a warning not to use the past or family history to make a diagnosis.
Incorrect
Headaches that are precipitated upon standing and relieved by lying flat occur following a lumbar puncture and these are referred to as post-LP headache, they are due to a CSF leak. A spontaneous CSF leak can occur, often following very trivial injury to the neck or back. Patients present with the identical headache to those seen after lumbar puncture, severe on standing and abolished by lying flat with the feet slightly elevated, if the head is slightly elevated it would be greatly relieved that may not be abolished. It is generally bilateral and worse in the occipital region and frontal region.
Chronic daily headache is a headache that is essentially present all day every day, fluctuating in severity from hour to hour day to day. It can get worse with exertion but it is not abolished by lying flat. It is usually bilateral frontal and occiptial and non throbbing.
Occioital neuralgia is a unilateral stabbing or shooting pain that last seconds at a time and can recur many times per dayBenign exertional headache can be of sudden onset although often there a mild headache that increases in severity as the exertion continues. The past and family history of migraine was a red herring totally irrelevant and put here as a warning not to use the past or family history to make a diagnosis.
-
Question 41 of 54
41. Question
A 29 year old female developed a sensation of instability in her head when she stood up followed 1 hour later by nausea without vomiting. Later that night she developed a pressure sensation behind both eyes that increased in severity overnight and was more severe on waking the following morning. When this pressure sensation reached its peak she experienced a sensation as if the right side of her face was sagging, her right eye felt as if it was on her right cheek. Once again she was nauseated without vomiting and there were no other symptoms. The whole episode resolved after 3 days. An MRI of the brain 1 week later was normal. A pregnancy test was -ve.
Correct
This patient had a past history of migraine aura without headache consisting of spreading photopsia. The distortion of body image is referred to as metamorphopsia. It is characteristic of the “Alice in Wonderland Syndrome. First described in 1955, Alice in Wonderland syndrome (AIWS) is a perceptual disorder characterised by distortions of visual perception (metamorphopsia), the body schema, and the experience of time. The name refers to Lewis Carroll’s well-known children’s book Alice’s Adventures in Wonderland,1 in which Alice feels (among other things) her body growing both larger and smaller. A 2016 review found 170 cases in the literature (Blom, J.D. Alice in Wonderland syndrome. A systematic review. Neurol Clin Pract. 2016 Jun; 6(3): 259–270).
The following table is taken from Bloms article
Incorrect
This patient had a past history of migraine aura without headache consisting of spreading photopsia. The distortion of body image is referred to as metamorphopsia. It is characteristic of the “Alice in Wonderland Syndrome. First described in 1955, Alice in Wonderland syndrome (AIWS) is a perceptual disorder characterised by distortions of visual perception (metamorphopsia), the body schema, and the experience of time. The name refers to Lewis Carroll’s well-known children’s book Alice’s Adventures in Wonderland,1 in which Alice feels (among other things) her body growing both larger and smaller. A 2016 review found 170 cases in the literature (Blom, J.D. Alice in Wonderland syndrome. A systematic review. Neurol Clin Pract. 2016 Jun; 6(3): 259–270).
The following table is taken from Bloms article
-
Question 42 of 54
42. Question
Category: Case 30A 42 year old female elite cyclist has persistent left buttock pain after a fall from her bicycle 2 months before being seen. She cannot sit because of the severe pain and tenderness in her buttock. A suspected diagnosis of referred pain from the lumbosacral spine was entertained even though the patient was not examined by the practitioner at the time. The MRI scan demonstrated a broad-based posterior disc protrusion at the L3-4 level and a broad-based disc protrusion at the L5-S1 level with narrowing of the exiting parameter and severe stenosis of both subarticular recesses and impingement on the S1 nerve root.
Examination revealed normal strength and reflexes, straight leg raising was 90° bilaterally, there was no sensory deficit in the lower limb. There was marked tenderness on firm palpation of the buttock below the issue, reproducing her pain. External rotation of the leg did not produce pain.Correct
It is said that more mistakes are made by not looking and not knowing, the clue was that she couldn’t sit on the buttock because it was so tender. Localised tenderness detected on examination points to local pathology it is not seen with referred pain. The radiological findings in the lumbosacral spine need to be ignored because such degenerative changes are very common and often asymptomatic. An ultrasound of the buttock did not reveal an explanation, I since learned that for deep tissue ultrasound is not useful. An MRI scan demonstrated a low-grade partial-thickness tear of the anti-margin of gluteus medius, the rectus femoris and mild tendinosis at the origin of the hamstring tendons and focal chondrosis near the chondrolabral junction
Elite female cyclists frequently suffer from secondary amenorrhoea and osteoporosis and thus a person this age could have suffered and impacted fractured neck of femur with a fall. Exacerbation of pain by external rotation is the clue to hip joint pathology.
The muscles of the buttock consists of the gluteus medius, gluteus maximus and gluteus minimus. These are supplied by the inferior gluteal nerve. These muscles are supplied by the superior and inferior gluteal nerve on the nerve root origin is L5-S1. In theory the possible nerve root compression at that level could cause pain in this region but it cannot explain localised tenderness.
Incorrect
-
Question 43 of 54
43. Question
Category: Case 31A 68-year-old man presents with five months of intermittent throbbing left temporal headaches. They would inevitably occur in the car when he was driving home from work at about 3:45 PM. The driving was stressful. The headaches were over the temple and above the year radiating into the occipital region. They would increase in severity very rapidly over one or two minutes and persist for about 30 minutes. The only other symptom he noticed was that his temporal artery was violated and pulsating but not tender. There was no past history or family history of migraine. His past history included diabetes, hypercholesterolaemia, gout, osteoarthritis of the cervical spine, gastro-oesophageal reflux, a perforated right tympanic membrane, peptic ulcer disease, ischaemic heart disease. His medications include Asterix, Diaformin, Dilthiazide, Lengout, Lopid, Metoprolol, Nexium, Nicorandil, Nordip, Olmetec, Phyiotens, Pravachol, Progout, Ticagrelor and Trajenta. His ESR was 27.
At the time of the consultation he was headache free on examination was completely normal.
Correct
Despite the absence of reddening and watering of the eye, blockage and ipsilateral nostril, miosis and ptosis the onset at exactly the same time of the day is typical for cluster headache. A dilated temporal artery has been described with cluster headache.
Temporal arteritis affects people over the age of 60, although the temporal artery may be dilated it is not pulsatile, it is thrombosis and tender. Patients with temporal arteritis often have associated scalp tenderness and jaw claudication (pain on chewing). Occipital neuralgia by definition must be confined to the distribution of fixable nerve, his pain affected the temple and occipital region supplied by the first division of the trigeminal nerve and the acceptable nerve. Occipital neuralgia Giselle antedating brief pain and does not last 30 minutes. Unilateral throbbing headache could be migraine but the onset at exactly the same time each day is not a feature of migraine. All though this could be tension headache whilst driving home in heavy traffic, tension headache is rarely throbbing in nature and when asked if he experienced this headache at the same time of the day when he was not driving he answered yes.Incorrect
-
Question 44 of 54
44. Question
Category: Case 3228-year-old woman has had blackouts since the age of 13 with a brief period of remission between the age of 20 and 22. The only warning is a pressure sensation in her head that increases in severity over 1 to 2 hours. Then she lies down on the couch and within 10 minutes the episodes commence. They consist of her being unresponsive, blinking repetitively and her eyes darting chaotically in all directions. These episodes last about five minutes but have lasted as long as 10 minutes. After the episodes she is unable to speak and is crying for a further 10-15 minutes. There is no stiffening of her limbs and there is no tongue biting or incontinence.
Correct
The correct and is nonepileptic seizures. There are many reasons for this diagnosis. First of all they are predictable, the warning symptom is far too long for an aura, the episodes last too long for absence or complex partial seizures and although confusion can occur after a complex partial seizure, the description of unresponsiveness with fluttering the eyelids is more in keeping with an absent seizure in which case the patient should return to normal immediately the seizure has finished and these occur without any warning whatsoever. Another clue is the eyes darting chaotically in all directions.
Many of the female patients that I have seen with nonepileptic seizures have been sexually assaulted and that was the case here. This poor lass has suffered sexual assault before the age of 13 from both her father and family friend. The treatment is not to give anticonvulsants to recommend psychological counselling.
Incorrect
-
Question 45 of 54
45. Question
Category: Case 33A 47 year old female has had several months of episodes of numbness and tingling affecting her left arm and leg. The sensory symptoms commence in her left axilla and then spread over several minutes to affect the entire left arm including the hand and then down the leg to the calf. Each episode lasts several minutes but less than 10 minutes and neither during or after these episodes of numbness and tingling is there any loss of function nor is there any other symptoms. What is the diagnosis?
Correct
This is a very typical case of migraine aura without headache. The sensory symptoms commencing one part of the body and spread very slowly over several minutes to affect other parts of the body. Although focal century seizures can spread from one part of the body the other the spread would be far more rapid. One can see paroxysmal symptoms in multiple sclerosis but they don’t spread in this manner. The classic paroxysmal symptoms with multiple sclerosis include trigeminal neuralgia and paroxysmal tonic seizures. They are often very brief on the matter of seconds which is too short for seizures and migraine without headache. In transient ischaemic attacks the symptoms usually are maximum at onset, only very, very rarely do they spread and there is usually a loss of function with a transient ischaemic attack.
Incorrect
-
Question 46 of 54
46. Question
Category: Case 34A 51-year-old female presents with numbness commencing two years ago in her right foot that gradually spread up the entire foot in the back of the leg to the calf associated with progressive weakness in the right leg. One year later she notices numbness affecting the lateral three digits the right hand that gradually spread to affect all the fingers the head and increasing weakness of the right arm over the 12 months. The patient had noticed a tight sensation in the back of the neck and a shooting pain up the back of her neck to her head on the left side. This pain is exacerbated by straining. She has had constipation but no urinary sphincter symptoms.
The nature and the distribution of the symptoms is used to localise the problem within the nervous system. Based on this patient’s symptoms where do you think the problem is?
Correct
This is a tricky case, the right-sided weakness point to a problem on the right side of the spinal cord below the level of the foramen magnum or on the left side of the brain above the level of the foramen magnum. The numbness suggests involvement of the spinothalamic tract this pathway crosses immediately after it enters to spinal cord. This means the problem has to be on the left side of the spinal cord or left side of the brain. Pain when it is due to a neurological problem is always a parallel of latitude. The neck pain with pain radiating into the occipital region indicates pain at the level of C2 on the left side. Thus the symptoms would suggest lesion at the upper end of the spinal cord lower end of the medulla i.e. at the level of the foramen magnum. An MRI scan demonstrated a haemangioma blessed over at the level of the foramen magnum extending down into the upper aspects of the cervical spine and also extending into the posterior fossa above the level the foramen magnum.
Incorrect
This is a tricky case, the right-sided weakness point to a problem on the right side of the spinal cord below the level of the foramen magnum or on the left side of the brain above the level of the foramen magnum. The numbness suggests involvement of the spinothalamic tract this pathway crosses immediately after it enters to spinal cord. This means the problem has to be on the left side of the spinal cord or left side of the brain. Pain when it is due to a neurological problem is always a parallel of latitude. The neck pain with pain radiating into the occipital region indicates pain at the level of C2 on the left side. Thus the symptoms would suggest lesion at the upper end of the spinal cord lower end of the medulla i.e. at the level of the foramen magnum. An MRI scan demonstrated a haemangioma blessed over at the level of the foramen magnum extending down into the upper aspects of the cervical spine and also extending into the posterior fossa above the level the foramen magnum.
-
Question 47 of 54
47. Question
Category: Case 35A 54-year-old man presents with a two year history of headaches. The headaches commence with a pressure sensation beneath the left eye and very rapidly evolved to a severe pain that spreads from the bridge of the nose up across the top of the left eyebrow to the left temple, reaching 10+/10 severity and lasting up to 45 minutes. They were occurring several times per day and could awaken him from his sleep at two hourly intervals. When he had these headaches the left side of his nose was blocked. An MRI scan demonstrated an empty sella, a CT scan of the sinuses demonstrated extensive left maxillary sinusitis without a fluid level and a polyp in the left frontal sinus. The neurological examination with unremarkable. His ESR was 20.
Which of the following is the most likely diagnosis
Correct
Cluster headache is classically unilateral and around the eye. A characteristic feature is that patients can almost set the clock to them, they can occur at the same time every day for weeks on end or occasionally in patients with chronic cluster headache for months or years. The pain is of rapid onset and excruciatingly severe, often leading to patients banging their head on the wall or walking around. They can be precipitated by alcohol and many patients stopped drinking alcohol during a bout of cluster headache. The other characteristic features include an ipsilateral blocknose, ipsilateral watering of the eye and a pathognomonic symptom is ptosis with miosis i.e. a temporary Horner’s syndrome. Temporal arteritis is not usually paroxysmal, A normal ESR does not exclude temporal arteritis particularly in the early stages, temporal arteritis usually occurs in patients over the age of 65-70 years of age. Many patients have radiological findings of sinusitis but unless there is a fluid level the sinusitis is unlikely to be a cause of headache. If you chose paroxysmal hemicrania that would not have been an unreasonable choice. Some regard paroxysmal hemicrania as a variant of cluster headache but the headaches of paroxysmal hemicrania are usually much shorter in duration (1-20 minutes, up to 240 minutes) and more frequent (one every second day up to 25 per day), they respond very well to indomethacin where as cluster headache does not. The dose of indomethacin may have to be fairly high.
Incorrect
Cluster headache is classically unilateral and around the eye. A characteristic feature is that patients can almost set the clock to them, they can occur at the same time every day for weeks on end or occasionally in patients with chronic cluster headache for months or years. The pain is of rapid onset and excruciatingly severe, often leading to patients banging their head on the wall or walking around. They can be precipitated by alcohol and many patients stopped drinking alcohol during a bout of cluster headache. The other characteristic features include an ipsilateral blocknose, ipsilateral watering of the eye and a pathognomonic symptom is ptosis with miosis i.e. a temporary Horner’s syndrome. Temporal arteritis is not usually paroxysmal, A normal ESR does not exclude temporal arteritis particularly in the early stages, temporal arteritis usually occurs in patients over the age of 65-70 years of age. Many patients have radiological findings of sinusitis but unless there is a fluid level the sinusitis is unlikely to be a cause of headache. If you chose paroxysmal hemicrania that would not have been an unreasonable choice. Some regard paroxysmal hemicrania as a variant of cluster headache but the headaches of paroxysmal hemicrania are usually much shorter in duration (1-20 minutes, up to 240 minutes) and more frequent (one every second day up to 25 per day), they respond very well to indomethacin where as cluster headache does not. The dose of indomethacin may have to be fairly high.
-
Question 48 of 54
48. Question
Category: Case 36A 65-year-old female presented with progressive wasting and weakness of the small muscles of both hands and a sense of weakness in both feet coming on gradually over many decades. She had also noted altered sensation on the lateral aspect of the left thigh in more recent years. Her past history included reflux oesophagitis, myocardial infarction, chronic active hepatitis, TIA affecting the left face, arm and leg, CREST syndrome, impaired fasting glucose and cholecystectomy.
The examination revealed mild weakness of neck flexion but no facial weakness, there was no frontal baldness or cataracts. There was weakness of wrist extension, finger extension and finger abduction bilaterally with associated wasting of the small muscles. The long flexors of the fingers were of normal strengths. All reflexes were preserved and there was no sensory deficit. In the lower limbs there was weakness of hip flexion bilaterally and mild weakness of dorsi flexion and plantarflexion of the toes bilaterally. All reflexes were preserved and the sensory examination was normal.
Her CK was normal as was her thyroid function. Her ESR was 54. There was a polyclonal increase in her gamma globulins. Her anti Ro was positive, her ANCA, MPO were negative her ANF was positive at a titre of 1:160. Her blood sugar was normal. Nerve conduction studies demonstrated normal motor conduction velocities but mild slowing and a reduction in the amplitude of teh sensory responses.Correct
The pattern of weakness does not reflect what one would see with bilateral ulnar nerve lesions of the elbow. Patient would have weakness of finger abduction and of the long flexors of the media two digits. A C8-T1 radiculopathy is also incorrect because these patients have weakness of finger abduction and weakness of flexion of the distal phalanges is all four digits, they do not have wrist extension weakness. Although the ANF was 1:160 this is borderline in a patient of this age and does not indicate a diagnosis of SLE. Her anti-Ro antibodies would be more in keeping with mild scleroderma and Crest syndrome. Patients with myotonic dystrophy can present with distal wasting and weakness but there were no symptoms to suggest myotonia, there was no frontal baldness, no cataracts and no diabetes. There is an extremely rare entity called distal myopathy and a very rare form called Welander distal myopathy has been reported to have associated sensory involvement. (reference Udd B. Distal myopathies. Curr Neurol Neurosci Rep. 2014;14:434).
Incorrect
The pattern of weakness does not reflect what one would see with bilateral ulnar nerve lesions of the elbow. Patient would have weakness of finger abduction and of the long flexors of the media two digits. A C8-T1 radiculopathy is also incorrect because these patients have weakness of finger abduction and weakness of flexion of the distal phalanges is all four digits, they do not have wrist extension weakness. Although the ANF was 1:160 this is borderline in a patient of this age and does not indicate a diagnosis of SLE. Her anti-Ro antibodies would be more in keeping with mild scleroderma and Crest syndrome. Patients with myotonic dystrophy can present with distal wasting and weakness but there were no symptoms to suggest myotonia, there was no frontal baldness, no cataracts and no diabetes. There is an extremely rare entity called distal myopathy and a very rare form called Welander distal myopathy has been reported to have associated sensory involvement. (reference Udd B. Distal myopathies. Curr Neurol Neurosci Rep. 2014;14:434).
-
Question 49 of 54
49. Question
Category: Case 37This is not a multiple choice question. This is two cases that emphasise the importance of ascertaining the exact distribution of pain in the limb.
The first case was a 77-year-old man who like most patients his age had low back pain. He presented with a three month history of bilateral thigh pain in the absence of any weakness or sensory symptoms. The pain was down the lateral aspect of both eyes, worse with standing, relieved by sitting or lying down and not exacerbated by walking. CT scan demonstrated bilateral impingement of the L 3-4 and L-5 nerve roots at their respective neural exit foraminal most marked the L5 level. A CT guided nerve root injection at the L5 level that was without benefit. The distribution of his pain did not represent the dermatomal pattern one would see with a radiculopathy. His pain is on the outside of both thighs between the hip and knee, this is the distribution of the lateral cutaneous nerve of the thigh. He was very tender confirm pressure over the lateral cutaneous nerve of the thigh beneath the inguinal ligament just medial to the anterior superior spine. His pain resolved following steel injections into the area where the lateral cutaneous nerve of the thymus compressed.
The second case was a 41-year-old man stop he had pain down the lateral aspect of his left thigh. There was a clicking sensation in his hip joint that produced a momentary pain but not the constant pain down his left thigh. Despite this an orthopaedic surgeon eventually did a total hip joint replacement without benefit. He subsequently developed a numbness down the lateral aspect of the thigh was found to have degenerative disease at the L 4-5 level and underwent an L 4-5 discectomy without benefit. This is not surprising because the distribution of the pain and numbness was not in the area of the L5 nerve root. The examination revealed altered sensation within the distribution of the lateral cutaneous nerve of the thigh and tenderness on firm palpation of the lateral cutaneous nerve of the thigh beneath the inguinal ligament just medial to the anterior superior iliac spine.
Correct
These two cases reiterate the importance of establishing the exact distribution of pain in the limb. One needs to ascertain whether the pain is in a joint or whether it represents soft tissue injury such as lateral epicondylitis. Most pain is not of neurological origin, however if it is it must be in the distribution of a nerve root or single nerve. Pain can occur in more than one nerve root in patients with brachial neuritis and lumbosacral neuritis.
Incorrect
These two cases reiterate the importance of establishing the exact distribution of pain in the limb. One needs to ascertain whether the pain is in a joint or whether it represents soft tissue injury such as lateral epicondylitis. Most pain is not of neurological origin, however if it is it must be in the distribution of a nerve root or single nerve. Pain can occur in more than one nerve root in patients with brachial neuritis and lumbosacral neuritis.
-
Question 50 of 54
50. Question
Category: Case 38A 76 year old man presents with a 2 year history of recurrent episodes occuring twice day. There is a snese of ants crawling on his skin that commences in his left foot and spreads over 2-3 minutes up his left side affecting his leg, truink and left arm, subsequently spreading across his shoulders inoto the right arm, trunk and leg and then resolves. The episodes occur mid morning and late evening awakening himn from his sleep if he retires to be early. Two year prior he had herpes simplex encephalitis affecting the left hemisphere and associated with haemorrhagic transformation. No seizures occured but he was placed on prophylactic Levetiracetam 500mg BD. He also suffered from hypertension and hypercholesterolaemaia. The neurological examination is normal.
Correct
Well done, a sensation of ants crawling on the skin is referred to as formication. The clue was that the episodes occured at exactly the same time every day, 2 hours after his dose of Levetiracetam (Keppra). I simply googled Levetiracetam and formication and found it was a recognised cause. His episodes ceased shortly after ceasoing the Levetiracetam. It was safe to stop the drug as he had never had seizures.
Incorrect
That is incorrect try again
-
Question 51 of 54
51. Question
Category: Case 39A 30 year old female, an accomplished trumpeter complains that she can no longer play her trumpet for more than 30 minutes. Air escapes out through her nose and she cannot generate enough pressure to play.She had been seen by 3 ENT surgeons and they all said there was nothing wrong.
what would you do?Correct
This case highlights severasl points. First of all when sympotmss are stereotyped ie the same each time they are very likely to be of organic origin.
Secondly seeing patients with intermittent symptoms at the time of their symptoms is more likely to establish the cause. If the ENT Surgeons had told her to play the trumpet until the symptoms developed they woould have seen she developed palatal incompetence with nasal escape.
Thirdly this is something I had nevere seen before, a not uncommon scenario so I searched the internet and found the diagnosis immediately.It is velopharyngeal incompetence a career ending move in 30% of musicians who generate high pressures inside their mouths is trumpet, clairent and oboe players.
The patient herself discovered a solution. She applied as swimmers nose peg to her nostrils!Incorrect
-
Question 52 of 54
52. Question
Category: Case 40A 49 year old female presents with pain and numbness in her right foot. The pain was under the 3rd toe,when she walked it felt like she was walking on a stone or pebble. Thid=s had resolved.
Some time later tshe develop tingling and subsequently numbness on the top of the right foot affecting the 1st and 2nd toes.
On examination There was altered sensation in the area of symptoms, but no weakness or reflex abnormality in the right leg.no tederness could be detected at the site of teh painCorrect
It is crucial to ascertain the exact distribution of sensory symptoms sa this provides the clue to the diagnosis. I consult the anatomy textbook firmly imprinted on my medial temporal lobe. Anyone can look up an anatomy textbook to see what nerve or nerve root supplies the area affected. Sensory symptoms would have to be in the distribution of a single nerve, multiple nerves (eg peripheral neuropathy), a nerve root or reflect involvement of the spinothalamic tract. In this case the distribution of the sensory symptoms pointed to the deep peroneal (fibular) nerve.
It is not sciatica as the pain and sensory symptoms should affect the same area relecting compression of a lumbar or sacral nerve root. The pain is not in the same area and is clealry unrelated. it wa almost certainly a Morton’s neuroma , although one cannot be entirely sure as the pain is usually between two adjacent digits and is oftne burning in nature. A Morton’s neuroma is thickening of the tissue around one of the nerves leading to the toes . It is not a peripheral neuropathy as the sensory symptoms are focal and not in a circumferential pattern affecting the tips of the toes upwards. It is not a common peroneal (fibular) nerve lesion at the neck of the fibula,as this casues altered sensation on a greater area on the top of the foot and often the lateral aspect of the shin,with or without a foot drop.
The sensory symptoms are in the distribution of the deep peroneal (fibular) nerve. A most unusual problem. Palpation along the course of the nerve detected tenderness and a lump 14 cm proiximally indicating a neuroma on the nerve.Incorrect
-
Question 53 of 54
53. Question
Category: Case 41A 65-year-old man has been experiencing recurrent funny turns. They are brief and almost identical every time. They occur at any time of the day and occasionally awaken him from his sleep. He is fully alert during these episodes. He experiences the sudden onset of a strange sensation in his body, he then notices that his hairs stand on and he develops goosebumps in his arms. The episodes last 30-60 seconds. He had two episodes whilst undergoing EEG monitoring and no abnormality was detected.
Correct
These episodes are brief, unpredictable, stereotyped and involve positive phenomena. This is the classic feature of seizures. The fact that he remains alert during these episodes means that they are simple-partial seizures. A normal EEG can be seen in patients with simple partial seizures. He had previously undergone a craniotomy for a meningioma. The goosebumps and piloereection(hairs standing on end) indicate involvement of the autonomic nervous system.
Incorrect
-
Question 54 of 54
54. Question
Category: Case 41A 56 year old man presents with severe burning pain in his right thigh that came on when he experienced severe back pain whilst lifitng a heavy object at work. He had suffered a similar back injury in the past with pain in his left buttock and down his leg to his shin with numbness affecting the inner aspect pof his left shin. He had a successful L4-5 hemidiscectomy. His MRI scan of his lumbar spine showed a disc cimpressing his right L4 nerve root. He had a right L4 hemidisctomy without any improvement in his symptoms.
What are the possible reasons why he did not improve?Correct
“The distribution of his burning pain was down the anterior aspect of his right thigh, clearly NOT in the distributon of the L4 nerve root and not in the pattern of an individual nerve or nerve root.Careful examination showed patchy alteration to pain and temperatuse senstation down the right side up about T10 and subtle weakness with increased reflexes and an upgoing plantar reflex on the left. He had absent right biceps and brachioralialsi reflexes but that is on the qworng side and these were also asymptomatic. He had a hemicord lesion on the left, because the motor pathway crosses at the level of the foramewn magnum, the so called Brown-Sequard syndrome. The absent reflexes were due to nerve root compression at the level of C5 and C6 on the right. His MRI revealed a thoraci cavenous malformation on the left side of his spinal cord at T9.
Incorrect
Although improvement in the neurological deficit cannot be gauranteed, surgery usually releives the pain in nerve root compression. Although the wrong operation could have been performed the corect answer is that thw wrong operation was performed. This mans sensory symptoms were well above the right L4 nerve root and represented involvement of the spinothalamic tract. See correct answer “wrong diagnosis for further details. The L4 nerve root “compression” was asymptomatic